[obm-l] Const. de triângulo

2024-01-13 Por tôpico Luís Lopes
Saudações, oi Anderson,

Soluções usando fórmulas servem para mostrar que o triângulo é construtível e 
qual é sua forma e tamanho. Já ajuda naquela parte - suponha o problema 
resolvido. Mas a construção procurada deverá ser feita usando as propriedades 
da figura.

Posso mandar no privado para quem se interessar as construções com as figuras 
que um correspondente me enviou. Esse que tem h_c/b como dado é bem 
interessante.

Agora o problema  pode ser resolvido de 3 ou mais maneiras. Com 
medianas é sempre bom pensar em simetrias e paralelogramos.

Luís


-- 
Esta mensagem foi verificada pelo sistema de antiv�rus e
 acredita-se estar livre de perigo.


=
Instru��es para entrar na lista, sair da lista e usar a lista em
http://www.mat.puc-rio.br/~obmlistas/obm-l.html
=


[obm-l] Const. de triangulo

2024-01-11 Por tôpico Luís Lopes
Mando outra mensagem pois reply não funciona. Pensei que estava claro. Notação 
padrão de triângulo. Construir os triângulos com R com os dados fornecidos.
h_a altura; m_a mediana; 
b+c soma dos lados AC e AB (vértices do triângulo);
h_c:b razão h_c/b



-- 
Esta mensagem foi verificada pelo sistema de antiv�rus e
 acredita-se estar livre de perigo.


=
Instru��es para entrar na lista, sair da lista e usar a lista em
http://www.mat.puc-rio.br/~obmlistas/obm-l.html
=


[obm-l] Const. de triangulo

2024-01-11 Por tôpico Luís Lopes
Vou mandar um texto bem carequinha.

h_a,m_a,h_c:b
b+c,h_a,h_b:h_c

Luís


-- 
Esta mensagem foi verificada pelo sistema de antiv�rus e
 acredita-se estar livre de perigo.


=
Instru��es para entrar na lista, sair da lista e usar a lista em
http://www.mat.puc-rio.br/~obmlistas/obm-l.html
=


[obm-l] Teste

2024-01-11 Por tôpico Luís Lopes
Mensagens não chegam.
-- 
Esta mensagem foi verificada pelo sistema de antiv�rus e
 acredita-se estar livre de perigo.


=
Instru��es para entrar na lista, sair da lista e usar a lista em
http://www.mat.puc-rio.br/~obmlistas/obm-l.html
=


[obm-l] f(f(x))=e^(-x)

2023-09-24 Por tôpico Luís Lopes
Minhas mensagens não chegam quando faço reply. Tenho sempre que começar uma 
nova. Segue a que mandei ontem, agora incluindo o Gugu (parece que é assim que 
ele gosta de ser chamado).

%%

Saudações,

Obrigado aos que responderam. É por aí, Ralph. Seu argumento é quase uma cópia 
do que veio no e-mail. Coloco aqui sem editá-lo. Vou encaminhar sua resposta ao 
correspondente (não o conheço). Obrigado novamente. 
Luís

===
Navegando pela Internet Youtube encontrei um desafio atinente
à composição de FRVR .A  F função seguinte:  (FoF)(X) = e^(-X)
Após mostrar que a função exponencial é decrescente  e^(-X)
Se X(FoF)(Y)   é verdadeiro!  FD.
Se Xhttp://www.mat.puc-rio.br/~obmlistas/obm-l.html
=


[obm-l] f(f(x))=e^(-x)

2023-09-23 Por tôpico Luís Lopes
Saudações,

Existe tal f? Se sim, qual seria?

Recebi um e-mail com esta pergunta, sem maiores detalhes. Pelo e-mail, tal f 
não existe. Problema encontrado pelo remetente no YouTube.

Luís


-- 
Esta mensagem foi verificada pelo sistema de antiv�rus e
 acredita-se estar livre de perigo.


=
Instru��es para entrar na lista, sair da lista e usar a lista em
http://www.mat.puc-rio.br/~obmlistas/obm-l.html
=


[obm-l] Comunicação

2023-09-17 Por tôpico Luís Lopes
Data: 25/08/2023
De: Priscila Santana 
Para: obm-l@mat.puc-rio.br
Assunto: [obm-l] Comunicação
Olá!

Existe algum grupo de discussão de questões olímpicas no WhatsApp?

Atte.


Priscila S. da Paz

Sauda,c~oes,

O Renato Madeira administra um. Ele pede somente que quem pedir
para se inscrever que se identifique cono membro desta lista.

Renato Madeira
WhatsApp: 55 21 99889 1516

Luís Lopes


-- 
Esta mensagem foi verificada pelo sistema de antivírus e
 acredita-se estar livre de perigo.



[obm-l] construir triângulos dados

2022-10-09 Por tôpico Luís Lopes
Sauda,c~oes,

O problema terá duas soluções (h_a=h=altura e d_a=d=bissetriz interna, k=d/h) se
0https://www.wolframalpha.com/input?i=0%3CB%3C%3Dpi%2F3%3B+1%3Ck%3C%5Csec%28B%2F2%29%3B+tan%5E2B%3D4%28k%5E2-1%29%2F%28k%5E4-4k%5E2%2B4%29

https://www.wolframalpha.com/input?i=pi%2F3%3CB%3Cpi%2F2%3B+1%3Ck%3C%5Ccsc%28B%29%3B+tan%5E2B%3D4%28k%5E2-1%29%2F%28k%5E4-4k%5E2%2B4%29

Obrigado.
Luís



-- 
Esta mensagem foi verificada pelo sistema de antivírus e
 acredita-se estar livre de perigo.



[obm-l] Eliminar parâmetro t

2020-11-16 Por tôpico Luís Lopes
Sauda,c~oes,

Num problema de encontrar o lugar geométrico do vértice
A de um triângulo, encontrei como valores das coordenadas
x_A e y_A as seguintes expressões:

A(x_A,y_A) com  x_A = N/D,  y_A=P/D, onde N=m(v^2+t^2);
D=t(1+m^2); P=m^2 v^2 - t^2.

Fora t, que tem que ser eliminado, todos os outros parâmetros
são fixos e conhecidos.

Não consegui fazer, obtendo sempre uma identidade 0=0.

Um amigo que já me ajudou nessas questões mandou a resposta,
obtida por computador. Para facilitar meus cálculos, tinha feito a=2v.
Daí o  na fórmula enviada:

-a^2 m + 4 m x^2 - 4 x y + 4 m^2 x y - 4 m y^2 = 0.

O lugar geométrico é uma hipérbole equilátera. O locus está correto.

Como fazer isso ? Outras eliminações mais difíceis que ele me enviou
eu nem tentaria fazer à mão. Mas essa não parecia difícil.
Como fazer ? Qual a técnica ? Deve haver uma para o computador e
casos complicados.

Luís



[obm-l] polinômio irredutível

2020-08-16 Por tôpico Luís Lopes
Sauda,c~oes,

Como provar que um polinômio f(x) tendo como coeficientes números inteiros
é irredutível se e somente se f(x+a) é irredutível para algum  inteiro ?

Luís




-- 
Esta mensagem foi verificada pelo sistema de antivírus e
 acredita-se estar livre de perigo.



RE: [obm-l] teste

2020-08-08 Por tôpico Luís Lopes
Sauda,c~oes,

Oi Carlos Victor,

Obrigado por responder. Acho que recebi sua mensagem pois vc a enviou
tanto para a lista quanto para mim diretamente. E do mesmo jeito, acho que
você vai receber minha resposta somente porque estou escrevendo
diretamente pra você, com cópia pros dois endereços da obm.puc.

Isso já acontece comigo há muito, muito tempo. No começo quando fazia reply,
agora quando mando uma mensagem nova. Às vezes mando as mensagens por
um outro email, e essas eu vejo chegarem no email que mandei. Mas não chegam
nesse daqui. E muito certamente não chegam na lista também.

Se por acaso alguém da lista receber esta mensagem e puder entrar em contato
com o administrador, ficaria muito grato.

Obrigado.

Luís





De: Carlos Victor 
Enviado: sábado, 8 de agosto de 2020 17:01
Para: obm-l@mat.puc-rio.br 
Cc: owner-ob...@mat.puc-rio.br ; Luís Lopes 

Assunto: Re: [obm-l] teste


Há muito tempo que os meus emails enviados também estão assim e não sei o 
motivo.

Carlos Victor

PS : este email não sei se chegará aos companheiros da lista

Em 08/08/2020 17:39, Luís Lopes escreveu:

Recebo as mensagens normalmente. Mas não tenho confirmação de
chegada ao grupo das que envio. E não aparecem nos arquivos também.

Mandei uma há umas 5 horas intitulada polinômio minimal.
Chegou? Alguém recebeu ?

Luís


--
Esta mensagem foi verificada pelo sistema de antivírus e
acredita-se estar livre de perigo.


-- 
Esta mensagem foi verificada pelo sistema de antivírus e
 acredita-se estar livre de perigo.



[obm-l] teste

2020-08-08 Por tôpico Luís Lopes
Recebo as mensagens normalmente. Mas não tenho confirmação de
chegada ao grupo das que envio. E não aparecem nos arquivos também.

Mandei uma há umas 5 horas intitulada polinômio minimal.
Chegou? Alguém recebeu ?

Luís


-- 
Esta mensagem foi verificada pelo sistema de antivírus e
 acredita-se estar livre de perigo.



[obm-l] construção geométrica

2020-06-10 Por tôpico Luís Lopes
Sauda,c~oes,

Recebi o seguinte problema:

Construir P no circuncírculo de um triângulo ABC dado
tal que PA+PB=PC.

Alguém saberia fazer ?

Obrigado.

Abs,
Luís



-- 
Esta mensagem foi verificada pelo sistema de antivírus e
 acredita-se estar livre de perigo.



[obm-l] construir triângulo dados , ,

2020-05-11 Por tôpico Luís Lopes
Sauda,c~oes,

d_a : bissetriz interna do vértice A ; ADa = d
e_a : bissetriz externa do vértice A; AEa = ea
m_b : mediana BMb = m
b:c=b/c=k

Os problemas  e  podem ser construídos
com régua e compasso usando o teorema das bissetrizes e as
propriedades da divisão harmônica. Como dica, ver o problema 7
na página 12 do livro Geometria II do Wagner/Morgado/M. Jorge de 1974.

Mas só consegui construir o triângulo ABC no caso 
usando álgebra.

Resolvendo as equações, obtive um valor para  que é construtível
com régua e compasso:

c=sqrt(4m2 + 2d2(k+1)2/k)/(k+2) = sqrt(4m2 + 2d2(k+1)2k-1)/(k+2)

Embora aceitável, estas soluções são muito rotineiras e não acrescentam
nada aos problemas, não usam nenhuma propriedade das figuras. Ou seja,
deixam a desejar do ponto de vista das construções geométricas propriamente.

Alguém poderia propor uma construção elegante, usando as propriedades da
figura e os teoremas da geometria ?

Como exemplo, considere o problema . Resolva-o das duas
maneiras e você verá o que quero dizer.

Dica: ver os resultados das seções 1.9, 2.8 e a figura da página 271 da
referência acima. Como mostrado no livro Geometria I dos mesmos
autores de 1990, página 74--75, MR=b-c.

Abraço,
Luís


-- 
Esta mensagem foi verificada pelo sistema de antivírus e
 acredita-se estar livre de perigo.



[obm-l] RE: Logaritmos e Sequência

2020-03-06 Por tôpico Luís Lopes
Sauda,c~oes, oi Maikel,

Escrevi três páginas sobre isso no livro
Manual das Funções Exponenciais e Logarítmicas.

Luís



De: owner-ob...@mat.puc-rio.br  em nome de Maikel 
Andril Marcelino 
Enviado: sexta-feira, 6 de março de 2020 01:26
Para: obm-l@mat.puc-rio.br 
Assunto: [obm-l] Logaritmos e Sequência


Boa noite, pessoal! Estou fazendo um trabalho. Meu orientador afirmou que havia 
uma maneira de introduzir o conceito de logaritmo com progressões A. e G.. Na 
minha graduação eu elaborei uma aula, que abordava progressões, porém era sobre 
propriedades de logaritmos. Algum ser humano tem ideia de como abordar o 
conceito de logaritmos por meio de sequências?


Atenciosamente,

Maikel Andril Marcelino
(84) 9-9149-8991 (Contato)
(84) 8851-3451 (WhatsApp)

--
Esta mensagem foi verificada pelo sistema de antivírus e
acredita-se estar livre de perigo.

-- 
Esta mensagem foi verificada pelo sistema de antivírus e
 acredita-se estar livre de perigo.



[obm-l]

2020-02-14 Por tôpico Luís Lopes
Minhas mensagens não estão chegando. Tento mais uma vez.

Sauda,c~oes,

Construir o triângulo (sinteticamente, sem (muita) álgebra)
com os dados acima. k é um número real (construtível) conhecido.

 Não sei se pode servir como aquecimento mas o problema
 me parece mais fácil.

Fonte: Il Problema Geometrico Dal compasso al Cabri.
Italo D'Ignazio e Ercole Supra.

O segundo aparece no Petersen também.

Os problemas  e  são casos particulares com k=1.

Abraços,
Luís


-- 
Esta mensagem foi verificada pelo sistema de antivírus e
 acredita-se estar livre de perigo.



[obm-l] construção geométrica

2020-01-19 Por tôpico Luís Lopes
Sauda,c~oes,

Envio o enunciado do problema tal como está no livro.

Construct a triangle, which shall have its vertex in a
given line, having a given base and a given difference
of the angles at the base.

Fonte: Julius Petersen, Methods and Theories for the
solution of Problems of Geometrical Constructions, 1927.
Problema 327, p.59.

Abraços,
Luís


-- 
Esta mensagem foi verificada pelo sistema de antivírus e
 acredita-se estar livre de perigo.



[obm-l] soma com cevianas que passam pelo circuncentro

2019-12-19 Por tôpico Luís Lopes
Sauda,c~oes,

Encontrei um link com a prova:

https://www.cut-the-knot.org/m/Geometry/CeviansThroughCircumcenter.shtml

Esse site é muito bom.

Eu conhecia a prova 3 mas não sabia que o triângulo tinha que ser acutângulo.
Para triângulo retângulo vale também, por verificação direta.

Aí comecei a rever a prova para triângulos obtusângulos e vi que
havia um problema com (B-C)=90º. Acho que para triângulos obtusângulos
a igualdade pode valer mas tem que ver para quais casos ela
não serve. Talvez (B-C) > 90º como (115º,15º,50º) e (B-C) < 90º
como (105º,45º,30º) satisfazem mas (B-C) = 90º como (120º,30º,30º)
não satisfaz. Isso precisaria de outra investigação.

Abraços,
Luís





-- 
Esta mensagem foi verificada pelo sistema de antivírus e
 acredita-se estar livre de perigo.



[obm-l] soma com cevianas que passam pelo circuncentro

2019-12-18 Por tôpico Luís Lopes
Sauda,c~oes,

Sejam AO_a, BO_B e CO_c as cevianas que passam pelo circuncentro.
O_a na reta do lado  etc.

Como provar que 1/AO_a + 1/BO_b + 1/CO_c = 2/R ?

Luís


-- 
Esta mensagem foi verificada pelo sistema de antivírus e
 acredita-se estar livre de perigo.



[obm-l] polinômio redutível ?

2019-11-05 Por tôpico Luís Lopes
Sauda,c~oes,

Considere o polinômio

p(x)[h,m,s] = 9x^4 + 12s x^3 + 2(8h^2 - 20m^2 - s^2)x^2 + 4s(4m^2 - s^2)x + 
(4m^2 - s^2)^2 .

Fiz alguns testes para ver se p(x) pode ter suas raízes construtíveis. Esse 
polinômio aparece
na construção do triângulo dados .

1) h=4sqrt(3); m=sqrt(21); s=13

p(x) = 9x^4 + 156x^3 - 410x^2 - 4420x + 7225 = (x-5)(3x+17)(3x^2 + 50x - 85)

2 positivas e 2 negativas

2) h=4sqrt(3); m=2sqrt(3); s=13

p(x) = 9x^4 + 156x^3 - 50x^2 - 6292x + 14641 = (3x^2 + 26x - 121)^2

1 positiva e 1 negativa raízes duplas

3) h=4sqrt(3); m=(3/2)sqrt(3); s=13

p(x) = 9x^4 + 156x^3 + 160x^2 - 7384x + 20164 = ? (Ax^2 + Bx + C)(Dx^2 + Ex + F)

irredutível  4 raízes complexas

x_1 = -13/3 - 2 i sqrt(7/3) - 1/2 sqrt(2044/9 + 208/3 i sqrt(7/3))

x_2 = -13/3 + 2 i sqrt(7/3) - 1/2 sqrt(2044/9 - 208/3 i sqrt(7/3))

x_3 = -13/3 - 2 i sqrt(7/3) + 1/2 sqrt(2044/9 + 208/3 i sqrt(7/3))

x_4 = -13/3 + 2 i sqrt(7/3) + 1/2 sqrt(2044/9 - 208/3 i sqrt(7/3))

4) h=12; m=7; s=13

p(x) = 9x^4 + 156x^3 + 6x^2 + 1404x + 729 = 3 (3x^4 + 52x^3 + 2x^2 + 468x + 
243) =
(3 x^2 - (8 sqrt(13) - 26) x + 27) (3 x^2 + (26 + 8 sqrt(13)) x + 27)

irredutível  2 raízes complexas 2 negativas

5) h=13; m=7; s=13

p(x) = 9x^4 + 156x^3 + 406x^2 + 1404x + 729 = 1/9 (-3 x - 2 i sqrt(39 sqrt(3) - 
49) + 6 sqrt(3) - 13) (-3 x + 2 i sqrt(39 sqrt(3) - 49) + 6 sqrt(3) - 13)   (3 
x - 2 sqrt(49 + 39 sqrt(3)) + 6 sqrt(3) + 13) (3 x + 2 sqrt(49 + 39 sqrt(3)) + 
6 sqrt(3) + 13)

= 1/9(9 x^2 - 36 sqrt(3) x + 78 x + 81)(9 x^2 + 36 sqrt(3) x + 78 x + 81)

irredutível  2 raízes complexas 2 negativas

Só testei para h,m,s > 0 mas se não errei nessas contas parece que podemos 
fatorar p(x)[h,m,s]  como

9x^4 + 12s x^3 + 2(8h^2 - 20m^2 - s^2)x^2 + 4s(4m^2 - s^2)x + (4m^2 - s^2)^2  = 
(Ax^2 + Bx + C)(Dx^2 + Ex + F) com os coeficientes A,B,... F construtíveis.

Daria para calcular os coeficientes dos dois polinômios do segundo grau em 
função de h,m,s ?

Luís




-- 
Esta mensagem foi verificada pelo sistema de antivírus e
 acredita-se estar livre de perigo.



[obm-l] Retas envolvendo uma parábola

2019-10-11 Por tôpico Luís Lopes
Sauda,c~oes,

Numa troca de mensagens sobre um procedimento de resolução do problema
"construir um triângulo ABC dados " apareceu a seguinte afirmação:

When two points move along two intersecting
straight lines at constant speeds (not necessarily equal)
line connecting them envelopes a parabola.

Alguém poderia me dar a prova ou referências sobre isso ?

Obrigado.

Luís



-- 
Esta mensagem foi verificada pelo sistema de antivírus e
 acredita-se estar livre de perigo.



[obm-l] problema de contagem

2018-11-04 Por tôpico Luís Lopes
Sauda,c~oes, oi Anderson,

Não precisa pensar nisso, é um problema de contagem.

Quero enunciar com palavras todos os problemas possíveis.

Exemplos:

(A,B,C) - dados os três ângulos
(A,B,a) - dados dois ângulos e o lado oposto a um deles
Assim (A,B,b) não conta.
(A,B,c) - dados dois ângulos e o lado oposto ao terceiro
(A,a,b) - dados um ângulo, o lado oposto ao ângulo e outro lado
(A,b,c) - dados um ângulo e os lados que formam este ângulo
(h_a,d_a,d_b) - altura e bissetriz interna partindo do mesmo vértice
e uma outra bissetriz interna
(d_c,d_b,h_b) - não conta, idêntico ao acima

Abraços,
Luís


-- 
Esta mensagem foi verificada pelo sistema de antivírus e
 acredita-se estar livre de perigo.



[obm-l] problema de contagem

2018-10-30 Por tôpico Luís Lopes
Sauda,c~oes,


Construir um triângulo dados três quaisquer dos seguintes elementos:


A,B,C - ângulos

a,b,c - lados

h_a,h_b,h_c - alturas

m_a,m_b,m_c - medianas

d_a,d_b,d_c - bissetrizes internas

e_a,e_b,e_c - bissetrizes externas

R - circunraio
r - inraio

r_a,r_b,r_c - exraios


Quantos problemas diferentes podem ser criados ?


Problemas do tipo (A,B,C) e (A,a,R) são considerados. Assim como

(A,B,a) e (A,B,c). Mas (A,B,a) e (A,B,b) contam como um só.


Abs,

Luís








-- 
Esta mensagem foi verificada pelo sistema de antivírus e
 acredita-se estar livre de perigo.



[obm-l] Const. triângulo dados "B=2C,a,b-c" e "B=2C,b,c"

2018-09-16 Por tôpico Luís Lopes
Sauda,c~oes,

Oi Claudio,

Obrigado.

< Com centro em A trace um círculo de raio c,
< intersectando o diâmetro do semi-círculo original em B.
De modo geral, o círculo (A,c) terá duas interseções
B1 e B2 com o diâmetro. E somente uma delas serve.
Trace a bissetriz (d1) de um deles (B1) e seja D1 a
interseção de d1 com AC. Se o triângulo B1CD1 é
isósceles, então B1 é a solução. Do contrário, B2.

Abs,
Luís


-- 
Esta mensagem foi verificada pelo sistema de antivírus e
 acredita-se estar livre de perigo.



[obm-l] Re: Const. triângulo dados "B=2C,a,b-c" e "B=2C,b,c"

2018-09-15 Por tôpico Luís Lopes
Sauda,c~oes,


Já que ninguém respondeu, a construção de ambos os problemas

é imediata sabendo-se que b^2=ac + c^2 .


A construção apresentada no FGM para o 1º problema

não precisou usar essa informação.


Abs,

Luís






De: owner-ob...@mat.puc-rio.br  em nome de Luís 
Lopes 
Enviado: sexta-feira, 14 de setembro de 2018 13:25:39
Para: obm-l@mat.puc-rio.br
Assunto: [obm-l] Const. triângulo dados "B=2C,a,b-c" e "B=2C,b,c"


Sauda,c~oes,


No livro do FGM de Trigonometria o 1º problema

tem uma construção somente por geometria.


Já o 2º encontrei num livro dos anos 50 que comprei

num sebo. O autor é Plácido Loriggio. Não tem a

construção nem sugestão. Procuro uma solução

puramente geométrica.


Abs,

Luís




--
Esta mensagem foi verificada pelo sistema de antivírus e
acredita-se estar livre de perigo.

-- 
Esta mensagem foi verificada pelo sistema de antivírus e
 acredita-se estar livre de perigo.



[obm-l] Const. triângulo dados "B=2C,a,b-c" e "B=2C,b,c"

2018-09-14 Por tôpico Luís Lopes
Sauda,c~oes,


No livro do FGM de Trigonometria o 1º problema

tem uma construção somente por geometria.


Já o 2º encontrei num livro dos anos 50 que comprei

num sebo. O autor é Plácido Loriggio. Não tem a

construção nem sugestão. Procuro uma solução

puramente geométrica.


Abs,

Luís




-- 
Esta mensagem foi verificada pelo sistema de antivírus e
 acredita-se estar livre de perigo.



[obm-l] Soma de Produtos de Termos em PA

2018-08-30 Por tôpico Luís Lopes
Sauda,c~oes, oi Claudio,

Seja S_{k-1} = (n-1)/(a1*an) = \frac{n-1}{a_1a_n}.

Para provar a recíproca escrevi

S_k = S_{k-1} + \frac{1}{a_n a_{n+1}} = \frac{n}{a_1a_{n+1}}

e cheguei a

n(a_{n+1} - a_n)=a_{n+1} - a_1 (*).

Fazendo a) n=2 e b) n=3 em (*) tem-se

a) a_3 + a_1 = 2a_2

b) a_4 + a_2 = 2a_3

Mas não consegui provar que a_{k+1} + a_{k-1} = 2a_k .

Usando (*) ou de outra maneira, como provar a recíproca ?

[]s
Luís



-- 
Esta mensagem foi verificada pelo sistema de antivírus e
 acredita-se estar livre de perigo.



[obm-l] caminho mínimo (suite 1)

2018-05-26 Por tôpico Luís Lopes
Realmente as mensagens não aparecem (?) quando
respondo na continuação da última recebida. Então
abro (recomeço) uma nova.

Sauda,c~oes, oi Pedro José,

A construção apresentada diz que OP tem que ser a
bissetriz do ângulo APB. Por quê ? Chega-se a isso
fazendo contas e interpretando o resultado geometricamente ?

Daí se U é o conjugado harmônico de O em relação a AB
e Gamma é o círculo de diâmetro OU, então o(s) ponto(s)
procurado(s) encontra-se na interseção de Gamma e o
círculo dado.


Seja MN um diâmetro de um círculo de centro O. Se A e B
são dois pontos neste diâmetro tais que M < A  < O < B < N ,
encontre o(s) ponto(s) P do círculo tal que o caminho APB
seja mínimo.
====

Sds,
Luís

-- 
Esta mensagem foi verificada pelo sistema de antivírus e
 acredita-se estar livre de perigo.



[obm-l] caminho mínimo (suite)

2018-05-23 Por tôpico Luís Lopes
Sauda,c~oes,


Começo nova mensagem pois àquelas que respondo não aparecem.


=


Boa tarde,


< Ponto do círculo ou da circunferência?
Circunferência.

< A ordenação que você menciona se refere ao ponto A estar entre
< M e O e o B estar entre O e N?
Isso.

Sds,
Luís

De: owner-ob...@mat.puc-rio.br <owner-ob...@mat.puc-rio.br> em nome de Pedro 
José <petroc...@gmail.com>
Enviado: quarta-feira, 23 de maio de 2018 19:49:26
Para: obm-l@mat.puc-rio.br
Assunto: [obm-l] Re: [obm-l] caminho mínimo

Boa tarde!
Ponto do círculo ou da circunferência?
A ordenação que você menciona se refere ao ponto A estar entre M e O e o B 
estar entre O e N?
Saudações,
PJMS

Em Qua, 23 de mai de 2018 15:18, Luís Lopes 
<qed_te...@hotmail.com<mailto:qed_te...@hotmail.com>> escreveu:
Sauda,c~oes,

Numa apostila do Curso Bahiense (Nº 13, Desenho
Geométrico) do Haroldo Manta (alguém aqui o conhece(u),
foi aluno dele ?) encontro o seguinte problema:

Seja MN um diâmetro de um círculo de centro O. Se A e B
são dois pontos neste diâmetro tais que M < A  < O < B < N ,
encontre o(s) ponto(s) P do círculo tal que o caminho APB
seja mínimo.

Luís

-- 
Esta mensagem foi verificada pelo sistema de antivírus e
 acredita-se estar livre de perigo.



[obm-l] caminho mínimo

2018-05-23 Por tôpico Luís Lopes
Sauda,c~oes,


Numa apostila do Curso Bahiense (Nº 13, Desenho

Geométrico) do Haroldo Manta (alguém aqui o conhece(u),

foi aluno dele ?) encontro o seguinte problema:


Seja MN um diâmetro de um círculo de centro O. Se A e B

são dois pontos neste diâmetro tais que M < A  < O < B < N ,

encontre o(s) ponto(s) P do círculo tal que o caminho APB

seja mínimo.


Luís


-- 
Esta mensagem foi verificada pelo sistema de antivírus e
 acredita-se estar livre de perigo.



mensagens não chegam (Era: [obm-l] Como calcular?)

2018-03-03 Por tôpico Luís Lopes
Sauda,c~oes,


Acho que a função "responder" está com problemas.

As mensagens que mando deste jeito não estão

aparecendo. E como tenho outro endereço de email,

vejo que algumas aparecem num e não aparecem no outro.


Decidi então criar uma totalmente nova, fora do encadeamento

das outras.


Espero que não esteja multiplicando o envio de mensagens

mas gostaria de ter certeza que estas duas últimas foram

realmente recebidas.


Abraços,

Luís


=

[obm-l] Como calcular?


Sauda,c~oes, oi Douglas,

Obrigado pelo link. Gostei muito do trabalho do Carlos Victor.

Aproveito para reenviar uma mensagem que mandei há alguns
dias e que acho que não chegou.
=


[obm-l] determine all pair of integers (x,y) such that

Sauda,c~oes,

Obrigado pelas respostas. Ideias bem legais
nas soluções.

Abraços,
Luís
**


-- 
Esta mensagem foi verificada pelo sistema de antivírus e
 acredita-se estar livre de perigo.



[obm-l] determine all pair of integers (x,y) such that

2018-02-24 Por tôpico Luís Lopes
1 + 2^x + 2^(2x+1) = y^2


Sauda,c~oes,


Recebi o problema acima de um outro grupo.

Como resolver ?


Abs,

Luís





-- 
Esta mensagem foi verificada pelo sistema de antivírus e
 acredita-se estar livre de perigo.



[obm-l] cadeia de logaritmos (suite)

2017-11-09 Por tôpico Luís Lopes
Eu acho que as mensagens que fazem parte deste assunto
(cadeia de logaritmos) não estão seguindo fazendo reply.
Mudo então o título do assunto e inicio um novo.

=
Sauda,c~oes, oi Bernardo,

Obrigado por responder.

Não sei provar a sua sugestão (e gostaria de ver a prova);
e pior, nem sei como usá-la.

Por sua mensagem e a do Tássio depreendo que o problema
faz sentido.

Retomo o que fiz.

Calculei a2 e encontrei 100 < a2 = 198 + log2 < 1000

Então 2 < a3=log(a2) < 3

0 < a4=log(a3) < 1

a5=log(a4) < 0

Logo a solução não é por aí.

Abraços,
Luís

-- 
Esta mensagem foi verificada pelo sistema de antivírus e
 acredita-se estar livre de perigo.



[obm-l] cadeia de logaritmos

2017-11-04 Por tôpico Luís Lopes
Sauda,c~oes,


Bom dia.


Não consegui resolver a questão abaixo. Como fazer ?


Abraços,

Luís



Considere o número N = ((100¹°°)¹°°)¨¨¹°° (ou seja, 100 elevado a 100, elevado 
a 100, ...), onde o número 100 aparece 100 vezes (incluindo a base). Seja a 
sequência definida como:

a1 = log N
a2 = log (a1)
a3 = log (a2)
...
a99 = log (a98)
a100 = log (a99)

Pode-se afirmar que a99 + a100 é igual a:

a) 102
b) 202
c) 102,3
d) 202,3
e) 2,3

Obs: se necessário, utilize log2 = 0,30.


-- 
Esta mensagem foi verificada pelo sistema de antivírus e
 acredita-se estar livre de perigo.



[obm-l] duas cordas paralelas

2017-10-24 Por tôpico Luís Lopes
Sauda,c~oes,


Sejam A e C dois pontos de um círculo Gamma=(O,R).

Construir duas cordas paralelas AB e CD tais que

AB+CD=2u, onde u é um comprimento dado.


Luís


-- 
Esta mensagem foi verificada pelo sistema de antivírus e
 acredita-se estar livre de perigo.



[obm-l] construção geométrica

2017-10-17 Por tôpico Luís Lopes
Sauda,c~oes,


Encontrei o problema abaixo num livro antigo de

Desenho Geométrico. Autor: Plácido Loriggio.


(MACK 57) Dados dois círculos tangentes de raios

iguais a 2cm e 5cm, respectivamente; construa uma

corda no círculo maior que tenha uma extremidade no

ponto de tangência e cujo segmento entre as duas

circunferências seja igual a 4cm.


Como fazer ?


Luís


-- 
Esta mensagem foi verificada pelo sistema de antivírus e
 acredita-se estar livre de perigo.



[obm-l] soma de tan^2

2017-09-16 Por tôpico Luís Lopes
Sauda,c~oes,


Bom dia.


Me mandaram a seguinte questão:


(1) Seja S = tan²(1º) + tan²(3º) + tan²(5º) + ... + tan²(89º), calcule o valor 
de S.


Como resolver ? Obrigado.


Abs,

Luís


-- 
Esta mensagem foi verificada pelo sistema de antivírus e
 acredita-se estar livre de perigo.



[obm-l] Cálculo de determinante

2017-03-01 Por tôpico Luís Lopes
Já mandei 2 ou 3 vezes esta mensagem para a lista.
Não sei por que ela não aparece. Tento novamente.

===
Oi, oi Douglas,

Sauda,c~oes,

Achei este problema legal e fiz uma busca por
"determinant of gcd matrix" no google.

Escolhi o link

http://math.stackexchange.com/questions/126/determinant-value-of-a-square-matrix-whose-each-entry-is-the-g-c-d-of-row-and-c

que me levou a

http://waset.org/publications/9996770/two-different-computing-methods-of-the-smith-arithmetic-determinant


< Obs: O resultado é MT bonito, uma potência de 2.
Verdade para n=1,2,….6. Fura para n=7.

Abs,
Luís

-- 
Esta mensagem foi verificada pelo sistema de antivírus e
 acredita-se estar livre de perigo.



[obm-l] {Disarmed} Enc: [EGML] Three circles in a triangle [1 Attachment]

2016-10-15 Por tôpico Luís Lopes
Sauda,c~oes,


Como construir (obter) o ponto D ?


A figura encontra-se no anexo (arquivo jpg).

Espero que ele a mostre.


Abs,

L.





[Attachment(s) from Antreas Hatzipolakis included below]

Nguyen Minh Tuan? wrote in a FB group (*):

Let ABC be a triangle and D a point such that the circles inscribed in the 
triangles ABD, BCD, and CAD pairwise touch each other. On lines BC, CA, AB, AD, 
BD, CD, denote the touching points by A1, B1, C1, A2, B2, C2, resp. Let lines 
B1C2 and B2C1 meet at E and let lines A1C2 and A2C1 meet at F . Show that the 
lines AF , BE and C1D are concurrent.

*

Let's replace E = B1C2 intersection B2C1 with Cb and F = A1C2 intersection A2C1 
with Ca.
Let C* be the point of concurrence of ACa , BCb and C1D.
Similarly A* and B*.

Are the triangles ABC, A*B*C* perspective?

APH

(*)

https://www.facebook.com/photo.php?fbid=1520551351291936=gm.1211999208873251=3


__._,_.___

Attachment(s) from Antreas Hatzipolakis | View attachments on the web 


1 of 1 Photo(s)

[$$1.jpg]
$$1.jpg



Reply via web 
post
*   Reply to sender 

   *   Reply to group 

*   Start a New 
Topic
*   Messages in this 
topic
 (1)

Check out the automatic photo album with 1 
photo(s)
 from this topic.
[$$1.jpg] 


[https://s.yimg.com/ru/static/images/yg/img/megaphone/1464031581_phpFA8bON]
Have you tried the highest rated email app?
With 4.5 stars in iTunes, the Yahoo Mail app is the highest rated email app on 
the market. What are you waiting for? Now you can access all your inboxes 
(Gmail, Outlook, AOL and more) in one place. Never delete an email again with 
1000GB of free cloud storage.

Visit Your 
Group

[Yahoo! 
Groups]
* Privacy * 
Unsubscribe * 
Terms of Use

.


__,_._,___

-- 
Esta mensagem foi verificada pelo sistema de antivírus e
 acredita-se estar livre de perigo.



Re: [obm-l] Re: [obm-l] nome de um quadrilátero

2016-08-10 Por tôpico Luís Lopes
Sauda,c~oes, oi Esdras,


Obrigado. Difícil imaginar isso pois rhombus

em inglês parece ser losango.


https://pt.wikipedia.org/wiki/Losango


Não me lembro de ter visto esse nome rombo.


Os livros didáticos usam esse nome para kite ?


Luís



De: owner-ob...@mat.puc-rio.br <owner-ob...@mat.puc-rio.br> em nome de Esdras 
Muniz <esdrasmunizm...@gmail.com>
Enviado: quarta-feira, 10 de agosto de 2016 21:13:26
Para: obm-l@mat.puc-rio.br
Assunto: [obm-l] Re: [obm-l] nome de um quadrilátero

Rombo.

Em 10 de agosto de 2016 17:43, Luís Lopes 
<qed_te...@hotmail.com<mailto:qed_te...@hotmail.com>> escreveu:

Sauda,c~oes,


Qual o nome em português para o

quadrilátero chamado de kite em inglês ?



https://en.wikipedia.org/wiki/Kite_(geometry)


Abs,

Luís


--
Esta mensagem foi verificada pelo sistema de antivírus e
acredita-se estar livre de perigo.



--
Esdras Muniz Mota
Mestrando em Matemática
Universidade Federal do Ceará



--
Esta mensagem foi verificada pelo sistema de antiv?rus e
acredita-se estar livre de perigo.

-- 
Esta mensagem foi verificada pelo sistema de antivírus e
 acredita-se estar livre de perigo.



[obm-l] nome de um quadrilátero

2016-08-10 Por tôpico Luís Lopes
Sauda,c~oes,


Qual o nome em português para o

quadrilátero chamado de kite em inglês ?



https://en.wikipedia.org/wiki/Kite_(geometry)


Abs,

Luís


-- 
Esta mensagem foi verificada pelo sistema de antivírus e
 acredita-se estar livre de perigo.



Re: [obm-l] soma binomial

2016-07-07 Por tôpico Luís Lopes
Sauda,c~oes, oi Anderson,


> Deve ter alguma forma de passar isso para uma função hipergeométrica

Deve ter. Tentei isso e só complicou.


> e ver se de fato tem solução fácil.

Ou melhor, uma solução esperta.

Pelo que sei do problema, deve ter. Vem do

Mathematical Reflections.


> Dei uma trapaceada, mas parece que o Wolfram Alpha não reconhece.

Ok. Nem pensei nisso. Mas acho que há programas capazes de

fornecer a forma fechada.


> Eu jogo diversos valores e isso tende a 1/3

Verdade. Fica S_n = 1/3 - ??


> - e o desejo de usar indução aumenta!

Verdade. A solução apresentada usa indução.

Mas acho nesse caso um pouco bastante artificial

pois o - ?? - acima veio do nada. A indução em si é fácil.


Na verdade comecei tentando S_n = \sum_{k=1}^n f(k)

com

f(k) = \frac{ k - 1 } { \binom{2k}{k} }

pois achei que era mais fácil com este f(k) do que com este aqui:

f(k) =  \frac{ 3k + 1 } { ( 2k + 1 ) \binom{2k}{k} } .


Neste deu pra calcular F(k) tal que F(k+1) - F(k) = \Delta F(k) = f(k)

e assim S_n = F(n+1) - F(1) = 1 - ?? .


Pro f(k) = \frac{ k - 1 } { \binom{2k}{k} } deve ter uma manipulação

binomial esperta pra obter o F(k) que não consigo ver.


Abs,

Luís



De: owner-ob...@mat.puc-rio.br <owner-ob...@mat.puc-rio.br> em nome de Anderson 
Torres <torres.anderson...@gmail.com>
Enviado: quinta-feira, 7 de julho de 2016 02:17:43
Para: obm-l@mat.puc-rio.br
Assunto: Re: [obm-l] soma binomial

Deve ter alguma forma de passar isso para uma função hipergeométrica e
ver se de fato tem solução fácil.

Dei uma trapaceada, mas parece que o Wolfram Alpha não reconhece. Eu
jogo diversos valores e isso tende a 1/3 - e o desejo de usar indução
aumenta!

Em 6 de julho de 2016 15:19, Luís Lopes <qed_te...@hotmail.com> escreveu:
> Sauda,c~oes,
>
>
> Alguém saberia como resolver (sem computador e indução) ?
>
> S_n = \sum_{k=1}^n f(k)
> com
> f(k) = \frac{ k-1 } { \binom{2k}{k} }.
>
> Abs,
> Luís
>
> --
> Esta mensagem foi verificada pelo sistema de antivírus e
> acredita-se estar livre de perigo.

--
Esta mensagem foi verificada pelo sistema de antiv?rus e
 acredita-se estar livre de perigo.


=
Instru??es para entrar na lista, sair da lista e usar a lista em
http://www.mat.puc-rio.br/~obmlistas/obm-l.html
Lista obm-l - Departamento de Matemática - 
PUC-Rio<http://www.mat.puc-rio.br/~obmlistas/obm-l.html>
www.mat.puc-rio.br
Lista obm-l. Existem pelo menos dois arquivos da lista obm-l. Um deles fica bem 
aqui, em http://www.mat.puc-rio.br/~obmlistas/obm-l.arquivo.html



=

-- 
Esta mensagem foi verificada pelo sistema de antivírus e
 acredita-se estar livre de perigo.



[obm-l] soma binomial

2016-07-06 Por tôpico Luís Lopes
Sauda,c~oes,

Alguém saberia como resolver (sem computador e indução) ?

S_n = \sum_{k=1}^n f(k)
com
f(k) = \frac{ k-1 } { \binom{2k}{k} }.

Abs,
Luís

-- 
Esta mensagem foi verificada pelo sistema de antivírus e
 acredita-se estar livre de perigo.



Re: [obm-l] Teorema de Varignon

2016-03-19 Por tôpico Luís
Sauda,c~oes,


Obrigado aos que escreveram.


Tudo começou com isso aqui


https://books.google.ca/books?id=mIT5-BN_L0oC=PA108_esc=y#v=onepage=false


Fala do Varignon e da reta de Newton.


Aí encontrei isso aqui.


http://www.academia.edu/1095647/Propriedades_para_visualização_da_reta_de_Newton


E há pouco me lembrei de algumas palavras do título que podia ser da Eureka.

Fiz a busca no site e encontrei


http://www.obm.org.br/export/sites/default/revista_eureka/docs/artigos/ponto_medio_cicero.pdf


Pronto. Tenho a referência em português.


Como o Sergio disse, muito pomposo citar uma referência em inglês ou

qualquer outra coisa diferente de português para tal teorema.


Luis



De: owner-ob...@mat.puc-rio.br <owner-ob...@mat.puc-rio.br> em nome de Sergio 
Lima <sergi...@smt.ufrj.br>
Enviado: sábado, 19 de março de 2016 17:42
Para: obm-l@mat.puc-rio.br
Assunto: Re: [obm-l] Teorema de Varignon

Oi, Luís,

Honestamente, não creio que esse resultado precise de uma citação.
Talvez não precise nem do nome pomposo de T. de Varignon.
Eu escreveria algo tipo "o que pode ser facilmente demonstrado com o
conceito de base média" e seguiria em frente.

Em todo caso, procurei nos Morgados e não achei.

Abraço,
Sergio

On Friday, March 18, 2016, Luís 
<qed_te...@hotmail.com<mailto:qed_te...@hotmail.com>> wrote:

Sauda,c~oes, oi Nehab, Marcelo,


Como disse, gostaria de ter uma referência em português.

Procurando algo no titio google sobre a reta de Newton-Gauss

caí no Varignon. E aí encontrei muita coisa em inglês e em

espanhol. Não procurei em francês pois na verdade quero

saber se por acaso teria visto o teorema em alguma publicação

em português.


Luis




De: owner-ob...@mat.puc-rio.br <owner-ob...@mat.puc-rio.br> em nome de Carlos 
Nehab <carlos.ne...@gmail.com>
Enviado: sexta-feira, 18 de março de 2016 18:26
Para: obm-l@mat.puc-rio.br
Assunto: Re: [obm-l] Teorema de Varignon


Titio google nao respondeu?

Em 18/03/2016 11:57, "Luís" <qed_te...@hotmail.com> escreveu:

Sauda,c~oes,


O teorema de Varignon é bem conhecido: os pontos médios dos lados

de um quadrilátero formam um paralelogramo.


Alguém conhece uma referência em português que o demostra ?

Não preciso da demonstração, só a citação.


Penso ter visto algo a respeito na RPM, Eureka, publicações do

IMPA ou livro de Geometria do Wagner/Morgado.


Abs,

Luís


--
Esta mensagem foi verificada pelo sistema de antivírus e
acredita-se estar livre de perigo.

--
Esta mensagem foi verificada pelo sistema de antiv?rus e
acredita-se estar livre de perigo.

--
Esta mensagem foi verificada pelo sistema de antivírus e
acredita-se estar livre de perigo.

--
Esta mensagem foi verificada pelo sistema de antiv?rus e
acredita-se estar livre de perigo.

-- 
Esta mensagem foi verificada pelo sistema de antivírus e
 acredita-se estar livre de perigo.



[obm-l] Teorema de Varignon

2016-03-19 Por tôpico Luís
Sauda,c~oes,


O teorema de Varignon é bem conhecido: os pontos médios dos lados

de um quadrilátero formam um paralelogramo.


Alguém conhece uma referência em português que o demostra ?

Não preciso da demonstração, só a citação.


Penso ter visto algo a respeito na RPM, Eureka, publicações do

IMPA ou livro de Geometria do Wagner/Morgado.


Abs,

Luís


-- 
Esta mensagem foi verificada pelo sistema de antivírus e
 acredita-se estar livre de perigo.



Re: [obm-l] Teorema de Varignon

2016-03-19 Por tôpico Luís
Sauda,c~oes, oi Nehab, Marcelo,


Como disse, gostaria de ter uma referência em português.

Procurando algo no titio google sobre a reta de Newton-Gauss

caí no Varignon. E aí encontrei muita coisa em inglês e em

espanhol. Não procurei em francês pois na verdade quero

saber se por acaso teria visto o teorema em alguma publicação

em português.


Luis




De: owner-ob...@mat.puc-rio.br <owner-ob...@mat.puc-rio.br> em nome de Carlos 
Nehab <carlos.ne...@gmail.com>
Enviado: sexta-feira, 18 de março de 2016 18:26
Para: obm-l@mat.puc-rio.br
Assunto: Re: [obm-l] Teorema de Varignon


Titio google nao respondeu?

Em 18/03/2016 11:57, "Luís" 
<qed_te...@hotmail.com<mailto:qed_te...@hotmail.com>> escreveu:

Sauda,c~oes,


O teorema de Varignon é bem conhecido: os pontos médios dos lados

de um quadrilátero formam um paralelogramo.


Alguém conhece uma referência em português que o demostra ?

Não preciso da demonstração, só a citação.


Penso ter visto algo a respeito na RPM, Eureka, publicações do

IMPA ou livro de Geometria do Wagner/Morgado.


Abs,

Luís


--
Esta mensagem foi verificada pelo sistema de antivírus e
acredita-se estar livre de perigo.

--
Esta mensagem foi verificada pelo sistema de antiv?rus e
acredita-se estar livre de perigo.

-- 
Esta mensagem foi verificada pelo sistema de antivírus e
 acredita-se estar livre de perigo.



Re: [obm-l] coordenadas do ortocentro

2016-02-23 Por tôpico Luís

Sauda,c~oes, 

Obrigado ao Carlos Gomes. Após o envio da mensagem 
fui ao site da RPM e fiz uma busca por Morgado e coordenadas 
e encontrei o artigo. Reflexo que já deveria ter adquirido. 

A fórmula acabou não sendo útil e fiz os cálculos naturais de 
retas perpendiculares e interseção de duas delas. Encontrei 
H=(-1/t, -t) com t = x_A x_B x_C . Não deu muito trabalho. 

> [AT]
> Basicamente, este resultado diz que se três pontos estão em uma
> hipérbole então seu ortocentro também estará.
Isso. Hipérbole equilátera. 

Essa parte acabou. E a recíproca ? Segue texto abaixo


Et pour poursuivre l'exercice, la question du dimanche soir : la réciproque 
était-elle correcte ? 
Si l'orthocentre d'un triangle dont les trois sommets se trouvent sur une 
hyperbole se trouve 
aussi sur cette hyperbole, celle-ci est-elle équilatère ?

Se os 4 pontos considerados encontram-se numa mesma hipérbole, a hipérbole é 
equilátera ? 
Questão mais interessante e talvez mais difícil. 

http://www.les-mathematiques.net/phorum/read.php?8,415494,415503 

Luís 



De: owner-ob...@mat.puc-rio.br <owner-ob...@mat.puc-rio.br> em nome de Anderson 
Torres <torres.anderson...@gmail.com>
Enviado: terça-feira, 23 de fevereiro de 2016 10:11
Para: obm-l@mat.puc-rio.br
Assunto: Re: [obm-l] coordenadas do ortocentro

Basicamente, este resultado diz que se três pontos estão em uma
hipérbole então seu ortocentro também estará.
Parece divertido! Dá para pressupor que os três pontos estão num
círculo unitário, e quem sabe adaptar um pouco...

Em 22 de fevereiro de 2016 17:22, Pedro José <petroc...@gmail.com> escreveu:
> Boa tarde!
>
> Não tentei simplificar. Mas considerando que os três pontos não sejam
> colineares.
>
> o triângulo com vértices: A(xa,ya); B(x,b,yb) e C(xc,yc).
>
> Temos que a reta l(C,Hc), onde Hc é o pé da altura relativa a C, é
> perpendicular a l(A,B) e C pertence a reta.
>
> Logo sua equação é : (xb-xa) x + (yb-ya) y = c1. Como c pertence a reta c1=
> (xb-xa) xc + (yb-ya)yc.
>
> Da mesma forma  a reta l(B,Hb), onde Hb é o pé da altura relativa a C, é
> perpendicular a l(A,C) e B pertence a reta.
>
> Logo sua equação é; (xc-xa) x + (yc-ya) y = c2. Como B pertence a reta c2 =
> (xc-xa) xb + (yc-ya)yb.
>
> Como H é l(C,Hc) interseção l(B,Hb) temos que H(xh,yh), onde Xh e yh é a
> solução do sistema abaixo:
>
> (xb-xa) x + (yb-ya) y = (xb-xa) xc + (yb-ya)yc (i)
> (xc-xa) x + (yc-ya) y = (xc-xa) xb + (yc-ya)yb.(ii)
>
> (xc-xa)*(i) - (xb-xa)* (ii) ==> [(xc-xa)*(yb-ya) - (xb-xa)*(yc-ya)]*y =
> (xb-xa)*(xc-xa)*(xc-xb) + (xc-xa)*(yb-ya)yc - (xb-xa)*(yc-ya)*yb.
>
> ==> y = [(xb-xa)*(xc-xa)*(xc-xb) + (xc-xa)*(yb-ya)yc - (xb-xa)*(yc-ya)*yb] /
> [(xc-xa)*(yb-ya) - (xb-xa)*(yc-ya)]
>
> e de forma análoga:
>
> x = [(yb-ya)*(yc-ya)*(yc-yb) + (yc-ya)*(xb-xa)xc - (yb-ya)*(xc-xa)*xb] /
> [(yc-ya)*(xb-xa) - (yb-ya)*(xc-xa)]
>
> Se os valores não forem literais. Melhor seria uma translação e uma rotação
> para que tenhamos três vértices da forma:
>
>
> (0,0) , (x2',0) e (x3',y3').
>
> Achar o H nesse sistema de coordenadas e depois efetuar a rotação inversa e
> a translação inversa das coordenadas obtidas para H.
>
> Espero que não tenha errado nada.
>
> Saudações,
> PJMS
>
>
> Em 21 de fevereiro de 2016 15:19, Carlos Gomes <cgomes...@gmail.com>
> escreveu:
>>
>> É na RPM 43 no artigo "Coordenadas para para os Centro do Triângulo"
>>
>> Se A,B e C são os vértices, H o ortogentro, e a,b e c são as medidas dos
>> ângulos internos do triângulo, então
>>
>> H=(A.tga+B.tgb+C.tgc)/(tga+tg+b+tgc)
>>
>> Cgomes.
>>
>>
>>
>> Em 21 de fevereiro de 2016 14:04, Luís <qed_te...@hotmail.com> escreveu:
>>>
>>> Sauda,c~oes,
>>>
>>>
>>> Sejam os pontos A=(x_A, 1/x_A), B=(x_B, 1/x_B) e C=(x_C, 1/x_C)
>>>
>>> vértices do triângulo ABC. Então H=(x_H, 1/x_H).
>>>
>>>
>>> Encontrei a prova deste resultado em diversos sites.
>>>
>>>
>>> Há algum (muito) tempo o Morgado publicou um artigo numa RPM
>>>
>>> dando as coordenadas dos centros notáveis do triângulo.
>>>
>>>
>>> Alguém sabe qual o volume ? E quais são as coordenadas do ortocentro ?
>>>
>>>
>>> Obrigado.
>>>
>>>
>>> Luís
>>>
>>>
>>>
>>> --
>>> Esta mensagem foi verificada pelo sistema de antivírus e
>>> acredita-se estar livre de perigo.
>>
>>
>>
>> --
>> Esta mensagem foi verificada pelo sistema de antivírus e
>>

[obm-l] coordenadas do ortocentro

2016-02-21 Por tôpico Luís
Sauda,c~oes,


Sejam os pontos A=(x_A, 1/x_A), B=(x_B, 1/x_B) e C=(x_C, 1/x_C)

vértices do triângulo ABC. Então H=(x_H, 1/x_H).


Encontrei a prova deste resultado em diversos sites.


Há algum (muito) tempo o Morgado publicou um artigo numa RPM

dando as coordenadas dos centros notáveis do triângulo.


Alguém sabe qual o volume ? E quais são as coordenadas do ortocentro ?


Obrigado.


Luís


-- 
Esta mensagem foi verificada pelo sistema de antivírus e
 acredita-se estar livre de perigo.



[obm-l] Re: Mostrar que Soma (k = 1, n) 1/P'(x_k) = 0

2016-02-19 Por tôpico Luís
Sauda,c~oes, 

Parece que não chegou. Mando novamente. 

Luís


De: Luís <qed_te...@hotmail.com>
Enviado: sexta-feira, 19 de fevereiro de 2016 14:35
Para: obm-l@mat.puc-rio.br
Assunto: Re: Mostrar que Soma (k = 1, n) 1/P'(x_k) = 0

Sauda,c~oes, oi Amanda,

Apesar de não conseguir uma solução, gostei deste problema.

Para n=2 e n=3 podemos ver que isso é verdade.

P[2](x)=(x - x_1) (x - x_2)
P[2]'(x) = (x - x_2) +  (x - x_1)
P[2]'(x_1) = (x_1 - x_2)
P[2]'(x_2) = (x_2 - x_1)

1/P[2]'(x_1) + 1/P[2]'(x_2) = 1/(x_1 - x_2) + 1/ (x_2 - x_1) = 0

P[3](x) = (x - x_1) (x - x_2) (x - x_3)
P[3]'(x) =  (x - x_2) (x - x_3) +  (x - x_1)  (x - x_3) +  (x - x_1) (x - x_2)
P[3]'(x_1) = (x_1 - x_2) (x_1 - x_3)
P[3]'(x_2) = (x_2 - x_1) (x_2 - x_3)
P[3]'(x_3) = (x_3 - x_1) (x_3 - x_2)

Soma (k = 1,3) 1/P[3]'(x_k) = 0

Mostrar assim para n >= 4 começa a ficar longo. Pensei então em indução.

P[n+1](x) = (x - x_(n+1)) Q[n](x)

P[n+1]'(x) = Q[n](x) + (x - x_(n+1)) Q[n]'(x)

Soma (k = 1,n) 1/Q[n]'(x)  = 0 (hipótese da indução)

Soma (k = 1,n+1) 1/P[n+1]'(x_k) = Soma (k = 1,n) 1 / [ (x_k - x_(n+1)) 
Q[n]'(x_k) ]  + 1 / Q[n](x_(n+1))

Parei aqui. Será que é possível manipular a expressão e chegar ao resultado ?

Luís



De: owner-ob...@mat.puc-rio.br <owner-ob...@mat.puc-rio.br> em nome de Merryl 
<sc...@hotmail.com>
Enviado: terça-feira, 16 de fevereiro de 2016 23:18
Para: obm-l@mat.puc-rio.br
Assunto: [obm-l] Mostrar que Soma (k = 1, n) 1/P'(x_k) = 0

Tentando mostrar isto, cheguei a uma expressão extremamente complicada. Podem 
ajudar?

Seja P um polinômio de grau n >= 2 tal que suas n raízes x_1, ... x_n sejam 
distintas duas a duas. Mostre que

Soma (k = 1, n) 1/P'(x_k) = 0

Obrigada

Amanda





=
Instru��es para entrar na lista, sair da lista e usar a lista em
http://www.mat.puc-rio.br/~obmlistas/obm-l.html
=
-- 
Esta mensagem foi verificada pelo sistema de antiv�rus e
 acredita-se estar livre de perigo.


=
Instru��es para entrar na lista, sair da lista e usar a lista em
http://www.mat.puc-rio.br/~obmlistas/obm-l.html
=


RE: [obm-l] Ajuda numa desigualdade.

2016-01-28 Por tôpico Luís
Sauda,c~oes, oi Douglas, 
Vou dar uma dica: faça a_(n+1) = ? e a_1=……=a_n = ?? Dai use G  <= A ( no caso 
G  < A ) .
Abs, L. 

Date: Thu, 28 Jan 2016 16:15:11 -0200
Subject: RE: [obm-l] Ajuda numa desigualdade.
From: profdouglaso.del...@gmail.com
To: obm-l@mat.puc-rio.br

Erro? Bom no meu celular acho que saiu as fórmulas todas fora de ordem rs
Em 28/01/2016 16:02, "Bruno Lira"  escreveu:









Primeiramente, tome
a função logaritmântica f(x) = ln(x) cujo domínio é o conjuntos dos
números reais maiores que ou igual a zero. Note que a função f é
injetora. Portanto, para provarmos que:




n n+1
( 1 + 1
)<  ( 1 +   1  
 )
(   n ) (
   n+1 )




basta provar que:




   (n)   (  n+1)
ln(  ( 1 + 1
)   )   <  ln( ( 1 +   1 )  )
   ( (   n ) )   ( ( n+1)  ) .




De fato, temos que:




   (n)( n+1)
ln(  ( 1 + 1
)  )   –   ln(  ( 1 +   1   )   )=
   ( (  n )  )((  n + 1)   ) 





   (n)(n+1)
ln(  ( n
+ 1 )  )   –   ln( (
n +  2 
) )=
   ( (n)  
)( (  n + 1  )) 





   (   2n  )   

ln(  (   n
+ 1  )  .  n+1
) 
; Das
propriedades de logaritmo.
   ( (n (n+2))   n+2 ) 





Daí:




   ( n   ) 
ln(  (  n^2
+ 2n + 1  )  . 
n+1 )
 

   ( (n^2 + 2n   )n+2) 






Comon^2 + 2n < n^2 + 2n + 1en+1 < n + 2temos
que:





n
 

(  n^2 +
2n + 1 )  . 
n+1<1

(n^2 + 2n)   n+2
E da injetividade da função f temos:





   ( n   ) ln( ( n^2 + 2n + 1 ) . n+1 )   <
ln(1)=0   ( (n^2 + 2n   )n+2)
Isto é:

   (n)(n+1)ln( ( 1 + 1 )  ) – ln( ( 1 + 1 ) 
 )<0   ( (  n )  )( ( n+1  )  )






Logo,





n n+1( 1 + 1 ) < ( 1 + 1 )(   n ) ( n+1 )   
   C.Q.D
P.S.: Se tiver algum erro me avisem por favor.
From: esdrasmunizm...@gmail.com
Date: Thu, 28 Jan 2016 12:18:03 -0300
Subject: Re: [obm-l] Ajuda numa desigualdade.
To: obm-l@mat.puc-rio.br

L = ((1+1/(n+1))^(n+1))/(1+1/n)^n = ((1 - 1/(n+1)²)^n)((n+2)/(n+1)) 
Use que (1 - x)^n > 1 - nx, Para x \in (0, 1)
L > (1 - n/(n+1)²)((n+2)/(n+1)) = ((n²+n+1)/(n²+2n+1))((n+2)/(n+1)) = 
(n³+3n²+3n+2)/(n³+3n²+3n+1) > 1.


Esse último termo é maior que 1.
Em 28 de janeiro de 2016 09:41, Douglas Oliveira de Lima 
 escreveu:
Opa Marcelo, muito obrigado mesmo, eu estou procurando uma solução daquelas 
tipodesigualdades, onde efetuamos uma estratégia para chegar no resultado, tipo 
uma daquelas que tu encontra no livro de combinatória do MOrgado(o problema das 
apostas).Mas valeu, se conseguir uma dessas me manda novamente por 
favor.AbraçoDouglas Oliveira
Em 28 de janeiro de 2016 01:26, Marcelo Salhab Brogliato  
escreveu:
Oi, Douglas, tudo bem?
Se provarmos que f(x) = (1 + 1/x)^x é estritamente crescente, então está 
provada sua desigualdade.
Uma maneira é fazer isso usando cálculo. Seja g(x) = ln(f(x)) = x ln(1 + 1/x). 
Assim, se provarmos que g(x) é estritamente crescente, então f(x) também será 
(exercício: prove essa afirmação).
g'(x) = ln(1 + 1/x) + x * (-1/x^2) / (1 + 1/x)  = ln(1 + 1/x) - (1/x) / (1 + 
1/x) = ln(1 + 1/x) - 1/(1+x)
Temos que mostrar que g'(x) > 0 para todo x.
Sabemos que ln(x) < x - 1, para x != 1. Aplicando essa desigualdade em 1/x, 
temos: ln(1/x) < 1/x - 1 => ln(x) > 1 - 1/x, para x != 1.
Aplicando a desigualdade acima em 1+1/x, temos: ln(1+1/x) > 1 - 1/(1 + 1/x) = 
(1/x) / (1 + 1/x) = 1/(1+x). Logo: ln(1+1/x) > 1/(1+x) => g'(x) > 0 para todo x 
(já que 1+1/x > 1).
Abraços,Salhab
2016-01-28 0:34 GMT-02:00 Douglas Oliveira de Lima 
:
Olá caros amigos, gostaria de uma ajuda na seguinte desigualdade 
(1+1/n)^n<(1+1/n+1)^(n+1), para n natural.
Agradeço desde já.







-- 
Esdras Muniz Mota
Mestrando em Matemática
Universidade Federal do Ceará



  
  

[obm-l] RE: [obm-l] Re: [obm-l] duas séries e um resultado

2016-01-14 Por tôpico Luís
Oi, oi Ralph, 
Obrigado pelo interesse e respostas. 

Você está certo. "Eles" fazem exatamente isso. 
Recebi o artigo onde a questão aparece: 
Almkvist., G e Berndt, B., "Gauss, Landen, Ramanujan, The A-G Mean, Ellipses, 
\pi, and the Ladies Diary". The American Mathematical Monthly, 95, 1988, pp. 
585--608. 
Obs.: o artigo dá a questão como um exemplo da falta de rigor predominante na 
época em que foi "resolvida". 
Abs, Luís 

From: ralp...@gmail.com
Date: Mon, 11 Jan 2016 17:27:52 -0200
Subject: [obm-l] Re: [obm-l] duas séries e um resultado
To: obm-l@mat.puc-rio.br

Pior que eu sei o que "eles" QUEREM que voce faca -- mas que estah errado. Eles 
QUEREM pensar assim:
sqrt(2)b=SUM (1/sqrt(k))b = SUM (1/sqrt(2k))
Entao quando voce faz (sqrt(2)-1)b, voce tem a soma dos inversos das raizes dos 
inteiros, da qual voce subtrai a soma dos inversos das raizes dos pares, 
ficando a soma dos inversos das raizes dos impares, que seria o a.
Mas, como eu disse, estah errado -- pelo menos no universo dos reais, b nao 
existe, nem sqrt(2)b, nem a.
Abraco, Ralph.

2016-01-11 17:18 GMT-02:00 Ralph Teixeira <ralp...@gmail.com>:
Bom, se eu entendi, do jeito que estah eh falso, porque nenhuma destas series 
converge!
(Bom, pelo menos nos reais... A menos que eles estejam em algum outro 
sistema...)

Abraco, Ralph.
2016-01-11 12:31 GMT-02:00 Luís <qed_te...@hotmail.com>:



Sauda,c~oes, 
Um bom 2016 para todos. 
Recebi o seguinte problema. 
a = \sum_{k=0}^\infty \frac{1}{\sqrt{2k+1}} e 
b = \sum_{k=1}^\infty \frac{1}{\sqrt{2k}}. 
Mostre que a / b = \sqrt{2} - 1. 
Abs, Luís 
  



  

[obm-l] duas séries e um resultado

2016-01-11 Por tôpico Luís
Sauda,c~oes, 
Um bom 2016 para todos. 
Recebi o seguinte problema. 
a = \sum_{k=0}^\infty \frac{1}{\sqrt{2k+1}} e 
b = \sum_{k=1}^\infty \frac{1}{\sqrt{2k}}. 
Mostre que a / b = \sqrt{2} - 1. 
Abs, Luís 
  

[obm-l] RE: [obm-l] Re: [obm-l] duas séries e um resultado

2016-01-11 Por tôpico Luís
Oi, oi Ralph, 
Concordo. Pensei então no seguinte problema: 
c_n = a_n / b_n. 
Mostre (será ??) que c = lim c_n =  \sqrt{2} - 1. 
a_n = \sum_{k=0}^n \frac{1}{\sqrt{2k+1}} e 
b_n = \sum_{k=1}^n \frac{1}{\sqrt{2k}}. 

From: ralp...@gmail.com
Date: Mon, 11 Jan 2016 17:18:01 -0200
Subject: [obm-l] Re: [obm-l] duas séries e um resultado
To: obm-l@mat.puc-rio.br

Bom, se eu entendi, do jeito que estah eh falso, porque nenhuma destas series 
converge!
(Bom, pelo menos nos reais... A menos que eles estejam em algum outro 
sistema...)

Abraco, Ralph.
2016-01-11 12:31 GMT-02:00 Luís <qed_te...@hotmail.com>:



Sauda,c~oes, 
Um bom 2016 para todos. 
Recebi o seguinte problema. 
a = \sum_{k=0}^\infty \frac{1}{\sqrt{2k+1}} e 
b = \sum_{k=1}^\infty \frac{1}{\sqrt{2k}}. 
Mostre que a / b = \sqrt{2} - 1. 
Abs, Luís 
  

  

[obm-l] RE: RES: [obm-l] relação trigonométrica

2015-10-22 Por tôpico Luís
Sauda,c~oes, oi Vitório, 

Ia mandar a mensagem abaixo quando vi a sua resposta. 
Vou ver o seu link em seguida. 

Minha mensagem param outro site:

===
Solved. 

It is known that in triangle ABC
tan ((B-C)/2) = (b-c)/(r+r_a) and tan(A/2)=r/(p-a) = r_a/p. 

Then r+r_a=(b+c) tan(A/2) and the result follows. 
===

Abs, 

Luís 



From: vitorio.si...@trf1.jus.br
To: obm-l@mat.puc-rio.br
Date: Thu, 22 Oct 2015 15:42:45 -0200
Subject: RES: [obm-l] relação trigonométrica
















Boa tarde Luis

 

Essa relação de Napier pode ser vista no
site http://math.tutorvista.com/trigonometry/napiers-analogy.html:

 

Abs,

Vitório









De: owner-ob...@mat.puc-rio.br
[mailto:owner-ob...@mat.puc-rio.br] Em nome de Luís

Enviada em: quinta-feira, 22 de
outubro de 2015 08:22

Para: obm-l@mat.puc-rio.br

Assunto: [obm-l] relação
trigonométrica



 



Sauda,c~oes, 



Considere o triângulo ABC com b>c e o ângulo 

D = (B-C)/2. 



Como provar que tan D = ((b-c)/(b+c)) cot(A/2) ? 



Abs, 

Luís 





-- 

Esta mensagem foi verificada pelo sistema de antivírus e 

acredita-se estar livre de perigo. 




--

Esta mensagem foi verificada pelo sistema de antivírus e 

 acredita-se estar livre de perigo.   
-- 
Esta mensagem foi verificada pelo sistema de antivírus e
 acredita-se estar livre de perigo.



[obm-l] relação trigonométrica

2015-10-22 Por tôpico Luís
Sauda,c~oes, 

Considere o triângulo ABC com b>c e o ângulo 
D = (B-C)/2. 

Como provar que tan D = ((b-c)/(b+c)) cot(A/2) ? 

Abs, 
Luís 

  
-- 
Esta mensagem foi verificada pelo sistema de antivírus e
 acredita-se estar livre de perigo.



[obm-l] números especiais OMERJ 2015

2015-10-15 Por tôpico Luís
Sauda,c~oes, 

Um número é dito especial se ele tem dois ou mais algarismos 
e é múltiplo da soma dos seus algarismos. Por exemplo, 12 é 
especial pois é múltiplo de 1 + 2 = 3. 

a) encontre três números especiais consecutivos; 

b) encontre quatro números especiais consecutivos. 

Fonte: OMERJ 2015 Nível 4 

Abs, 
Luís 

  
-- 
Esta mensagem foi verificada pelo sistema de antivírus e
 acredita-se estar livre de perigo.



[obm-l] construir triangulo

2015-04-25 Por tôpico Luís
Sauda,c~oes, 

Construir com régua e compasso um triângulo ABC dados 

1) b+c, r_a, r_b 

2) b+c, h_b, m_c 

Abs, 
Luís 

  
-- 
Esta mensagem foi verificada pelo sistema de antivírus e
 acredita-se estar livre de perigo.



[obm-l] RE: pentágono cíclico

2015-04-25 Por tôpico Luís
Sauda,cões, 

 A mensagem abaixo não chegou. Deve ter sido pelo anexo. 

Quem quiser receber o arquivo escreva-me pedindo. 

Luís 


From: qed_te...@hotmail.com
To: obm-l@mat.puc-rio.br
Subject: FW: pentágono cíclico
Date: Sat, 25 Apr 2015 21:21:38 +




Sauda,cões, 

A questão faz parte de uma investigação mais profunda. 

Espero que o pdf em anexo siga. 

Luís 

 Date: Sat, 25 Apr 2015 20:53:54 +0100
 From: amon...@ull.es
 To: qed_te...@hotmail.com
 Subject: RE: pentágono cíclico
 
 Estimado Luís
 
 Izán Péraz me planteó el siguiente problema:
 
 Calculate the area of a cyclic polygon with five sides which is convex 
 and whose length sides are 13, 13, 5+12*sqrt(3), 20*sqrt(3), 
 -5+12*sqrt(3)
 
 Después de consultar (pag. 528):
 
 Areas of Polygons Inscribed in a Circle
 Author(s): David P. Robbins
 Source: The American Mathematical Monthly, Vol. 102, No. 6 (Jun. - Jul., 
 1995), pp. 523-530
 
 
 Con mis cálculos obtengo 640.859
 
 Me interesa la construcción  de un tal pentágono, para confirmar mi 
 solución.
 
 Un saludo
 Angel
 
 El 2015-04-25 02:20, Luís escribió:
  Caro Angel,
  
  Mandei sua pergunta para um grupo do qual faço parte
  e a pessoa respondeu com uma outra pergunta:
  
  Construir, tipo, com regua e compasso? Ou, num sentido mais teorico e
  geral?
  
   Bom, creio que seja construir com régua e compasso.
  Mas não sei em que isso pode influir na questão.
  
  Uma pergunta minha: a ordem dos lados tem que ser mantida?
  Ou qualquer permutação é permitida ?
  
  Saludos,
  Luis
  
  Date: Fri, 24 Apr 2015 10:31:14 +0100
  From: amon...@ull.es
  To: qed_te...@hotmail.com
  Subject: pentágono cíclico
  
  Estimado, Luís
  
  ¿Es posible construir un pentágono inscrito en una circunferencia
  cuyos
  lados a1,a2,a3,a4,a5 tiene longitudes: 13, 13, 5 + 12*Sqrt[3],
  20*Sqrt[3], -5 + 12*Sqrt[3]?
  
  Un saludo
  Angel

  
-- 
Esta mensagem foi verificada pelo sistema de antivírus e
 acredita-se estar livre de perigo.



[obm-l] FW: pentágono cíclico

2015-04-24 Por tôpico Luís
Sauda,c~oes, 

Alguém saberia responder ? 

Abraços, 
Luís 

 Date: Fri, 24 Apr 2015 10:31:14 +0100
 Subject: pentágono cíclico
 
 Estimado, Luís
 
 ¿Es posible construir un pentágono inscrito en una circunferencia cuyos 
 lados a1,a2,a3,a4,a5 tiene longitudes: 13, 13, 5 + 12*Sqrt[3], 
 20*Sqrt[3], -5 + 12*Sqrt[3]?
 
 Un saludo
 Angel
  
-- 
Esta mensagem foi verificada pelo sistema de antivírus e
 acredita-se estar livre de perigo.



[obm-l] uma colinearidade

2014-09-19 Por tôpico Luís
Sauda,c~oes, 
Bom dia. 
Como provar que M=Q' e N=P' ? Continue a ler. 
Desenhe um circulo phi_1 e uma secante d com interseções U e V. Então UV é 
uma corda de phi_1. 
Desenhe um circulo phi_2 de centro V e raio b de modo que phi_1 e phi_2 se 
intersectam em P e Q. 
Trace as retas r=(P,U) e s=(Q,U)  e sejam M e N as interseções de r e s com 
phi_2.  M=r \cap phi_2 e N=s \cap phi_2. 
Sejam P' e Q' as reflexões de P e Q na secante d. Como d é um diâmetro de 
phi_2, P' e Q' estão em phi_2. 
Mas fazendo esta figura com o Geogebra percebi que M=Q' e N=P' e não preciso 
construir os pontos P' e Q'. 
Assim, a construção de ABC dados A,b,d_c, onde d_c é bissetriz interna de C 
fica um pouco mais leve. 
Aqui phi_1 seria o arco capaz de A sobre o segmento (corda) D_cC=d_c e P e Q 
seriam os dois vértices A1 e A2 do triângulo. 
Depois disto tudo, minha pergunta: como provar que M=Q' e N=P' ? 
Abs, Luís 
  
-- 
Esta mensagem foi verificada pelo sistema de antivírus e
 acredita-se estar livre de perigo.



[obm-l] coordenadas do incentro

2014-05-25 Por tôpico Luís
Sauda,c~oes, 
Numa RPM antiga o Morgado apresenta as coordenadas do incentro. 
Alguém saberia dizer qual foi o Volume? 
[]'s Luis 
  
-- 
Esta mensagem foi verificada pelo sistema de antivírus e
 acredita-se estar livre de perigo.



Re: [obm-l] ensinando tabelas verdade

2014-04-14 Por tôpico Luís Junior
Pedro José,

Vc trabalha na área de petróleo?


2014-04-14 16:21 GMT-03:00 Pedro José petroc...@gmail.com:

 Boa tarde!

 Não tenho texto pronto. Mas, é um pouco mais complicado que

 *e , ou.*

 p

 q

 P == q

 V

 V

 V

 V

 F

 F

 F

 V

 V

 F

 F

 V


 P (F) e Q (F ou V, tanto faz) == P ==Q (V)

 Exemplo: 2 = 4 == qualquer homem voa (V)

 Embora entenda que a  melhor forma de analisar a veracidade é verificando
 o que a negativa é.

 p e ^ ~q (não q)


 Para o exemplo acima:

 2 = 4 e Existe pelo menos um homem que não voa (F e V) == (F) se a
 negativa é F, assertiva é V.

 x^2 pertence 2 |N == x pertence a 2 |N.

 Analise a negativa.

 x^2 pertence a 2 |N e x  pertence a 2|N +1

 x^2 pertence a 2|N == x^2 ≡ 0 mod 2.  e x ≡ 1 mod2 (absurdo), pois se  x
 ≡ 1 mod2 temos que  x^2 ≡1  mod 2 (conservação da multiplicação)


 Então só temos p (F) e q (V) ou p (V) e q (F), pois 2 pertence a |P
 (conjunto dos primos).


 Ou poder-se-ia analizar Existe pelo menos um x^2 pertencente a 2|N e x
 pertence a 2 |N + 1. Isso é falso.


 x pertence a 2|N+1 == existe k pertencente a |N | x = 2k+1 == x^2 = 4k^2
 + 4*K + 1= 2 (2k^2+2*K), pelo fechamento da adição, multiplicação e
 potência em \N temos que Existe s = (2k^2+2*K) pertencente a 2|N, logo x^2
 pertence a 2\N+1.


 Espero que lhe ajude.


 Saudações,

 PJMS





 Em 20 de abril de 2014 15:28, Hermann ilhadepaqu...@bol.com.br escreveu:

  Ensinar tabela verdade,
 é fácl para os conectivos e e ou
 mas alguém tem uma dica de como ensinar a lógica da tabela verdade da
 condicional p-q.


 Abraços
 Hermann

 ps: se tiverem um texto pronto de alguém e quiserem mandar para o meu
 email, agradeço.

 --
 Esta mensagem foi verificada pelo sistema de antivírus e
 acredita-se estar livre de perigo.



 --
 Esta mensagem foi verificada pelo sistema de antivírus e
 acredita-se estar livre de perigo.


-- 
Esta mensagem foi verificada pelo sistema de antiv�rus e
 acredita-se estar livre de perigo.



Re: [obm-l] Questão difícil de combinatória - Campeonato Cearense de Futebol

2014-03-21 Por tôpico Luís Eduardo Háteras
Ola douglas,
Obrigado pela resolucao, mas nao estou concordando com a ultima 
expressao:[C(12,2).C(10,2).C(8,2).C(6,2).C(4,2).C(2,2)]/6! 
-[C(10,2).C(8,2).C(6,2).C(4,2).C(2,2)]/5! 
Pois quando vc escreve isso esta retirando os casos que fortaleza joga com 
ceara,mas o problema nao quer isso, eu interpretei que o problema esta 
interessado no ultimo jogo do primeiro turno que tem que ser fortaleza e ceara, 
entao, a ideia que vc usou (acredito) nao corresponde a realidade do problema...

Enviado via iPad

 Em 21/03/2014, às 10:51, douglas.olive...@grupoolimpo.com.br escreveu:
 
 Em 20.03.2014 23:38, Luís Eduardo Háteras escreveu:
 
 Sabendo-se que o campeonato cearense de futebol é disputado por 12 clubes, 
 entre os quais fortaleza e ceará.
 (A,B,C,D,E,F,G,H,I,J,FORTALEZA E CEARÁ)
 Determine a quantidade de maneiras de ocorrer o primeiro turno,
 PRIMEIRO TURNO 6 GRUPOS DE 2 EX: 
 (A,B);(C,D);(E,FORTALEZA);(F,CEARÁ);(G,H);(I,J) 
 COMO VEMOS FORTALEZA E CEARÁ NÃO ESTÃO JUNTOS
 de modo que o confronto entre fortaleza e ceara seja apenas na rodada final. 
 ENTENDI QUE ELES SÓ JOGARIAM JUNTOS NO FINAL E QUE SÓ INTERESSA A RESPOSTA 
 PARA O PRIMEIRO TURNO.
 Assim é só pegar 12 times e dividir em 6 grupos de 2 e retirar dessas 
 escolhas aquelas em que fortaleza e ceará estejam juntos.
 [C(12,2).C(10,2).C(8,2).C(6,2).C(4,2).C(2,2)]/6! 
 -[C(10,2).C(8,2).C(6,2).C(4,2).C(2,2)]/5! =Resposta
 
 -- 
 Esta mensagem foi verificada pelo sistema de antivírus e 
 acredita-se estar livre de perigo.
  
 
  
 
 -- 
 Esta mensagem foi verificada pelo sistema de antivírus e 
 acredita-se estar livre de perigo.

-- 
Esta mensagem foi verificada pelo sistema de antiv�rus e
 acredita-se estar livre de perigo.



[obm-l] Questão difícil de combinatória - Campeonato Cearense de Futebol

2014-03-20 Por tôpico Luís Eduardo Háteras
Sabendo-se que o campeonato cearense de futebol é disputado por 12 clubes, 
entre os quais fortaleza e ceará. Determine a quantidade de maneiras de ocorrer 
o primeiro turno, de modo que o confronto entre fortaleza e ceara seja apenas 
na rodada final. 
-- 
Esta mensagem foi verificada pelo sistema de antivírus e
 acredita-se estar livre de perigo.



RE: [obm-l] R^2=(BC^2+AH^2)/4

2014-02-20 Por tôpico Luís
Sauda,c~oes, 

Obrigado Carlos Victor. 

Lembrei-me depois que tudo isso está mostrado 
no livro de Geometria do Morgado. Mas a mensagem 
pra lista já havia chegado. 

Abraços, 
Luís 


Date: Mon, 17 Feb 2014 18:43:25 -0300
Subject: Re: [obm-l] R^2=(BC^2+AH^2)/4
From: victorcar...@globo.com
To: obm-l@mat.puc-rio.br

Sim Luís, Você pode encontrar essa relação em vários livros de geometria que 
fale sobre a reta de Euler,   que passa  pelo circuncentro, ortocentro e 
baricentro, ok ?Abraços

Carlos Victor

Em 17 de fevereiro de 2014 18:33, luiz silva luizfelipec...@yahoo.com.br 
escreveu:

Essa relação é valida em um triangulo qualquer ?


Abs
Felipe

 
 
  
   Em Segunda-feira, 17 de Fevereiro de 2014 15:49, Carlos Victor 
victorcar...@globo.com escreveu:
  
  Oi Luís,Apesar do enunciado não falar, H é o ortocentro do triângulo, ok 
?AbraçosCarlos Victor

Em 16 de fevereiro de 2014 22:33, luiz silva luizfelipec...@yahoo.com.br 
escreveu:

AH é a altura relativa à BC?   

Em Sábado, 15 de Fevereiro de 2014 17:30, Carlos Victor 
victorcar...@globo.com escreveu:

Oi Luís,digitei errado.Onde está AM lê-se  AH, ok ?Desculpe  o engano...
Carlos
  Victor
Em 15 de fevereiro de 2014 16:53, Carlos Victor victorcar...@globo.com 
escreveu:


Oi Luís,Seja M o ponto médio de BC  e O o circuncentro do triângulo ABC. 
Prove inicialmente que AM= 2.OM e aplique Pitágoras no triângulo OMC, por 
exemplo.



Daí sai legal a relação que tu queres, ok ?Para provar que AM = 2.OM , pense no 
alinhamento que existe entre o circuncentro, ortocentro e baricentro... .



AbraçosCarlos  VictorEm 13 de fevereiro de 2014 13:13, Luís 
qed_te...@hotmail.com escreveu:






Sauda,c~oes, 



Como provar a relação abaixo? 



R^2=(BC^2+AH^2)/4 



Imaginei colocar os pontos B,C,H com as seguintes coordenadas: 



B=(0,0)  C=(a,0)  H=(h,y_H) A=(h,y_A)



Daí a gente obtém o ponto H_c=(x,y) com régua e compasso e 



em seguida o ponto A. O circuncentro (O) é calculado e finalmente R. 



As contas não são legais com papel e lápis. Alguém poderia dar as 



coordenadas dos pontos A e (O) usando um programa de cálculo simbólico ? 



Obrigado. 



Luís 



  --
Esta mensagem foi verificada pelo sistema de antivírus e 
 acredita-se estar livre de perigo.




--
Esta mensagem foi verificada pelo sistema de antivírus e 
 acredita-se estar livre de perigo.
  --
Esta mensagem foi verificada pelo sistema de antivírus e 
 acredita-se estar livre de perigo.


--
Esta mensagem foi verificada pelo sistema de antivírus e 
 acredita-se estar livre de perigo.


  
--

Esta mensagem foi verificada pelo sistema de antivírus e 

 acredita-se estar livre de perigo.




--

Esta mensagem foi verificada pelo sistema de antivírus e 

 acredita-se estar livre de perigo.   
-- 
Esta mensagem foi verificada pelo sistema de antivírus e
 acredita-se estar livre de perigo.



[obm-l] R^2=(BC^2+AH^2)/4

2014-02-13 Por tôpico Luís
Sauda,c~oes, 
Como provar a relação abaixo? 
R^2=(BC^2+AH^2)/4 
Imaginei colocar os pontos B,C,H com as seguintes coordenadas: 
B=(0,0)  C=(a,0)  H=(h,y_H) A=(h,y_A)
Daí a gente obtém o ponto H_c=(x,y) com régua e compasso e em seguida o ponto 
A. O circuncentro (O) é calculado e finalmente R. 
As contas não são legais com papel e lápis. Alguém poderia dar as coordenadas 
dos pontos A e (O) usando um programa de cálculo simbólico ? 
Obrigado. 
Luís 
  
-- 
Esta mensagem foi verificada pelo sistema de antivírus e
 acredita-se estar livre de perigo.



RE: [obm-l] duas identidades

2014-01-22 Por tôpico Luís
Sauda,c~oes, oi Bernardo, 

Obrigado. 

É verdade. E a segunda também tem um typo. 
O revisor da época comeu mosca. 

Abs, 
Luís 

 Date: Tue, 21 Jan 2014 15:09:15 -0200
 Subject: Re: [obm-l] duas identidades
 From: bernardo...@gmail.com
 To: obm-l@mat.puc-rio.br
 
 2014/1/21 Luís qed_te...@hotmail.com:
  Sauda,c~oes,
 
  Como mostrar que
 
  x^(2n) - 1 = (x^2-1)(x^(2n)+x^(2n-1)++1)=(x^2-1) X
  \prod_{k=1}^{n-1} (x^2 - 2x cos(k\pi/n) + 1)
 
  e
 
  x^(2n+1) = (x+1)(x^(2n)-x^(2n-1)++1)=(x+1) X
  \prod_{k=1}^{n} (x^2 - 2x cos((2k-1)/(2n+1)) + 1)
 
 Olhe para as raízes complexas desses polinômios, e faça pares de
 raízes conjugadas. Vou fazer o primeiro:
 
 x^(2n) - 1 = 0 = x = exp(2 pi i * k /2n), para k = 0, 1, 2, … (2n -
 1). Separe k = 0 e k = n, que dão as raízes x = 1 e x = -1, sobram as
 raízes exp( +- 2 pi i * k / 2n) para k = 1, 2, … n-1 (usando que tudo
 é periódico módulo 2n !!). Seja w = exp(2 pi i / 2n), agrupando os
 fatores (x - w^k) e (x - w^(-k)) temos
 (x^2 - (w^k + w^(-k))x + 1) = (x^2 - 2 cos(k * 2 pi/2n) x + 1).
 
 Obs: a fatoração intermediária está errada, deveria começar em x^(2n -
 2), para dar o grau certo.
 
 Abraços,
 -- 
 Bernardo Freitas Paulo da Costa
 
 -- 
 Esta mensagem foi verificada pelo sistema de antivírus e
  acredita-se estar livre de perigo.
 
 
 =
 Instruções para entrar na lista, sair da lista e usar a lista em
 http://www.mat.puc-rio.br/~obmlistas/obm-l.html
 =
  
-- 
Esta mensagem foi verificada pelo sistema de antivírus e
 acredita-se estar livre de perigo.



[obm-l] duas identidades

2014-01-21 Por tôpico Luís
Sauda,c~oes, 
Como mostrar que 
x^(2n) - 1 = (x^2-1)(x^(2n)+x^(2n-1)++1)=(x^2-1) X\prod_{k=1}^{n-1} (x^2 - 
2x cos(k\pi/n) + 1) 
e 
x^(2n+1) = (x+1)(x^(2n)-x^(2n-1)++1)=(x+1) X\prod_{k=1}^{n} (x^2 - 2x 
cos((2k-1)/(2n+1)) + 1) 
Fonte: Mathematics Magazine March-April 1955 p.235 
Abs, Luis 
  
-- 
Esta mensagem foi verificada pelo sistema de antivírus e
 acredita-se estar livre de perigo.



[obm-l] forma geral da conica

2014-01-13 Por tôpico Luís
Sauda,c~oes, 
Seja a cônica dada pela equação 
Ax^2 + Bxy + Cy^2 + Dx + Ey + F = 0 (B/=0)
Como expressar os parâmetros da cônica (foco, centro, diretriz etc) em função 
dos parâmetros da equação ? 
Estou perguntando simbolicamente pois numericamente eu tenho estas informações 
com o Wolfram Alpha, por exemplo. 
Fiz um teste com 5x^2 + 2xy - 5y^2 + 6x + 8y + 12 = 0 
e soh não vi claramente apresentada a diretriz. 
Luis 
  
-- 
Esta mensagem foi verificada pelo sistema de antivírus e
 acredita-se estar livre de perigo.



RE: [obm-l] soma da Eureka

2013-12-31 Por tôpico Luís



Sauda,c~oes, 
Muito bom, Marcos. Obrigado. 
Pra terminar esta série de msgs, gostaria de tratar do problema 6 na p. 38, 
S(1921) = f(1) + .. + f(1921) para f(k) = 1/(sqr(k) + sqr(k^2 - 1))
Encontrei S(1921) = (sqr(2)/2)(sqr(1922) + sqr(1921) - 1). 
Esta certo? 
Luis 

Date: Mon, 30 Dec 2013 20:34:20 -0200
Subject: Re: [obm-l] soma da Eureka
From: mffmartine...@gmail.com
To: obm-l@mat.puc-rio.br

Na linha seguinte:
* {1/2 . sum{k = 2}^{100} [-1/k + 1/(k - 1)]}
Em segunda-feira, 30 de dezembro de 2013, Marcos Martinelli escreveu:

Uma pequena correção na escrita (quinta linha):
* = 1/2 . f(100) +1/2 . sum{k = 2}^{100} 1/(k^2 - k + 1)


Em segunda-feira, 30 de dezembro de 2013, Marcos Martinelli escreveu:

A gente pode considerar f(k) = (k + 1)/(k^2 + k + 1). 


Podemos mostrar a seguinte relação: 1/(k^4 + k^2 + 1) = 1/2 . [(k + 1)/(k^2 + k 
+ 1) - (k - 1)/(k^2 - k +1)] = 1/2 . [f(k) - f(k - 1) + 1/(k^2 - k +1)] .

Assim, a soma que queremos é tal que: sum{k = 1}^{100} 1/(k^4 + k^2 + 1) = [1/2 
. sum{k = 1}^{100} f(k)] - [1/2 . sum{k = 1}^{100} f(k - 1)] + [1/2 . sum{k = 
1}^{100} 1/(k^2 - k + 1)] = 1/2 . f(100) +1/2 . sum{k = 1}^{100} 1/(k^2 - k + 
1)  1/2 . f(100) + {1/2 . sum{k = 1}^{100} [-1/k + 1/(k - 1)]} = 1/2 . f(100) 
+ 1/2 . (1 - 1/100).



Agora, basta mostrarmos que: 1/2 . f(100) + 1/2 . (1 - 1/100)  1/2 = 
101/10101 + 1 - 1/100  1 = 101/10101  1/100 = 10100  10101 (V). c.q.d






--

Esta mensagem foi verificada pelo sistema de antivírus e 

 acredita-se estar livre de perigo.
  
-- 
Esta mensagem foi verificada pelo sistema de antivírus e
 acredita-se estar livre de perigo.



RE: [obm-l] soma da Eureka

2013-12-30 Por tôpico Luís
Sauda,c~oes, 
Obrigado Marcos. 
No problema 8, f(k) = 1/(k^4 + k^2 + 1). 
Conheço uma forma fechada para g(k) = k/(k^4 + k^2 + 1). 
Como f(k) = g(k) e \sum g(k)  1/2, então  \sum f(k)  1/2. 
Alguém tem outra solução ? 
Luis 

Date: Sun, 29 Dec 2013 22:26:08 -0200
Subject: Re: [obm-l] soma da Eureka
From: mffmartine...@gmail.com
To: obm-l@mat.puc-rio.br

f(x) + f(1 - x) = a^x/(a^x + sqr(a)) + a^(1 - x)/[a^(1 - x) + sqr(a)] = 
a^x/(a^x + sqr(a)) + a/(a + a^x . sqr(a)) = a^x/(a^x + sqr(a)) + sqr(a)/(a^x + 
sqr(a)) = 1.


Em domingo, 29 de dezembro de 2013, Luís escreveu:



Oi, oi Marcos, 
Verdade. O problema 4 tem uma solução parecida: f(x) + f(1/x) = 1. 
E o problema 5 na p. 38 ? f(x) = a^x/(a^x + sqrt(a)). 
Deve ter uma solução usando os argumentos vistos nestas duas últimas soluções. 
Alguma dica? 
Luis 

Date: Sun, 29 Dec 2013 18:20:29 -0200

Subject: Re: [obm-l] soma da Eureka
From: mffmartine...@gmail.com
To: obm-l@mat.puc-rio.br


Para resolver o problema proposto, repare que: f(x) + f(1 - x) = 2/(4^x + 2) + 
2/[4^(1 - x) + 2] = 2/(4^x + 2) + 4^x/(2 + 4^x) = 1.


Em domingo, 29 de dezembro de 2013, Luís escreveu:



Sauda,c~oes, 
Adaptando o problema 3 da p. 37 da Eureka 37, existiria ?? uma forma fechada 
para a soma 
S(n) = a_1 + . + a_n para a_k = \frac{2}{4^k + 2}


Ou também, como fazer o problema proposto ? 
Bom ano para todos. 
Luis 
  
--

Esta mensagem foi verificada pelo sistema de antivírus e 

 acredita-se estar livre de perigo.




--

Esta mensagem foi verificada pelo sistema de antivírus e 

 acredita-se estar livre de perigo.   
--

Esta mensagem foi verificada pelo sistema de antivírus e 

 acredita-se estar livre de perigo.




--

Esta mensagem foi verificada pelo sistema de antivírus e 

 acredita-se estar livre de perigo.   
-- 
Esta mensagem foi verificada pelo sistema de antivírus e
 acredita-se estar livre de perigo.



[obm-l] soma da Eureka

2013-12-29 Por tôpico Luís
Sauda,c~oes, 
Adaptando o problema 3 da p. 37 da Eureka 37, existiria ?? uma forma fechada 
para a soma 
S(n) = a_1 + . + a_n para a_k = \frac{2}{4^k + 2}
Ou também, como fazer o problema proposto ? 
Bom ano para todos. 
Luis 
  
-- 
Esta mensagem foi verificada pelo sistema de antivírus e
 acredita-se estar livre de perigo.



RE: [obm-l] soma da Eureka

2013-12-29 Por tôpico Luís
Oi, oi Marcos, 
Verdade. O problema 4 tem uma solução parecida: f(x) + f(1/x) = 1. 
E o problema 5 na p. 38 ? f(x) = a^x/(a^x + sqrt(a)). Deve ter uma solução 
usando os argumentos vistos nestas duas últimas soluções. 
Alguma dica? 
Luis 

Date: Sun, 29 Dec 2013 18:20:29 -0200
Subject: Re: [obm-l] soma da Eureka
From: mffmartine...@gmail.com
To: obm-l@mat.puc-rio.br

Para resolver o problema proposto, repare que: f(x) + f(1 - x) = 2/(4^x + 2) + 
2/[4^(1 - x) + 2] = 2/(4^x + 2) + 4^x/(2 + 4^x) = 1.


Em domingo, 29 de dezembro de 2013, Luís escreveu:



Sauda,c~oes, 
Adaptando o problema 3 da p. 37 da Eureka 37, existiria ?? uma forma fechada 
para a soma 
S(n) = a_1 + . + a_n para a_k = \frac{2}{4^k + 2}

Ou também, como fazer o problema proposto ? 
Bom ano para todos. 
Luis 
  
--

Esta mensagem foi verificada pelo sistema de antivírus e 

 acredita-se estar livre de perigo.




--

Esta mensagem foi verificada pelo sistema de antivírus e 

 acredita-se estar livre de perigo.   
-- 
Esta mensagem foi verificada pelo sistema de antivírus e
 acredita-se estar livre de perigo.



RE: [obm-l] funcao implicita e geogebra

2013-10-05 Por tôpico Luís Lopes
Sauda,c~oes, oi Bernardo, 

Bom, o que dizer? Muito obrigado, Bernardo!! 

  Continuo sem saber como calcular a equação que fornece
  os pontos extremos (max e min) da curva 
Agora sei. :) Pelo menos usando o WAlpha. 

 Se eu entendi o problema, você quer achar 
o(s) ponto(s) desta curva com a maior ordenada possível.
Exato. No intervalo 0 x10. 

 Agora, note que P(x(t),y(t)) = 0 em todos os pontos da curva, logo
 para todos os t da sua parametrização. Daí:
 dP/dx(x(t), y(t))*dx/dt + dP/dy(x(t), y(t))*dy/dt = 0. 
Comentário fundamental para a continuação. 

 Agora, você tem que achar as soluções do sistema {P(x,y) = 0,
 dP/dx(x,y) = 0}. No seu caso particular, dP/dx é de grau 2, logo você
 pode escrever y em função de x, e substituir na equação de grau 3...
No decorrer do pensamento vi que era por aí. Mas as contas 
se anunciavam pesadas. 

Os tais discriminantes são o caso particular do resolvente 
de um polinômio e de sua derivada. 
Bom, pode estar resumido mas estou satisfeito. 
Já sei o que são discriminantes neste contexto. 
E o melhor, o WA me dá eles. Coloquei o polinômio 
que dá a curva CC e ele me retornou a equação 
que já me haviam enviado. Daí calculei os pontos 
extremos da curva. Problema resolvido. 

O fator que sobra deve ser o polinômio de grau 4 que
passaram pra você.
Exato. 

Com isso o problema de construir o triângulo ABC dados 
A,a+b,h_a está resolvido e discutido.  O caso numérico 
estudado é cos A=11/14 e a+b=10. 

Se h_a  y_max  o problema não tem solução; 
se h_a=y_max um só triângulo satisfaz; 
se 0  h_a  y_max dois triângulos satisfazem. 

Deixo um problema com vocês: achar o lugar geométrico 
do vértice A dados o ângulo do vértice A e a diferença 
a-b. Seria a (nova) curva CC. 

Abs, 
Luís 






 Date: Sat, 5 Oct 2013 01:25:40 -0300
 Subject: Re: [obm-l] funcao implicita e geogebra
 From: bernardo...@gmail.com
 To: obm-l@mat.puc-rio.br
 
 2013/10/4 Luís Lopes qed_te...@hotmail.com:
  Sauda,c~oes,
 Oi Luís,
 
  Continuo sem saber como calcular a equação que fornece
  os pontos extremos (max e min) da curva mas talvez a
  teoria se encontre nos livros que tratam das Curvas Algébricas Planas.
 Exato, mas não necessariamente desta forma.
 
 Você tem uma equação implícita P(x,y) = 0, onde P é um polinômio (de
 grau 3) nas duas variáveis. Se eu entendi o problema, você quer achar
 o(s) ponto(s) desta curva com a maior ordenada possível.
 
 Para começar, pense que a curva definida pela equação P(x,y) = 0 é
 bonitinha (sem pontos duplos, sem singularidades, etc). Assim, imagine
 uma parametrização local da curva: (x(t), y(t)). Assim, pontos de
 máximo (e mínimo, ou de inflexão) são dados por dy/dt = 0.
 
 Agora, note que P(x(t),y(t)) = 0 em todos os pontos da curva, logo
 para todos os t da sua parametrização. Daí:
 dP/dx(x(t), y(t))*dx/dt + dP/dy(x(t), y(t))*dy/dt = 0.
 
 Se dy/dt = 0, temos então que dP/dx(x(t), y(t)) * dx/dt = 0. Como a
 curva é lisa (e sem singularidades, pontos duplos, etc, etc), dx/dt
 não pode ser = 0. Daí, os pontos de máximo também satisfazem uma
 equação suplementar, dP/dx(x,y) = 0.
 
 Agora, você tem que achar as soluções do sistema {P(x,y) = 0,
 dP/dx(x,y) = 0}. No seu caso particular, dP/dx é de grau 2, logo você
 pode escrever y em função de x, e substituir na equação de grau 3...
 
 Mas, em geral, existe uma teoria para achar as soluções de sistemas de
 equações simultâneas, que são os resolventes, cf
 http://en.wikipedia.org/wiki/Resultant e
 http://en.wikipedia.org/wiki/Sylvester_matrix, em particular a parte
 de aplicações a interseções do resultante. Essencialmente, eles
 substituem uma equação na outra, de forma algorítmica, e dão
 diretamente a equação que todas (e não metade, como seria o caso se
 você pegasse cada uma das soluções da eq do segundo grau) as abscissas
 possíveis y satisfazem. Os tais discriminantes são o caso particular
 do resolvente de um polinômio e de sua derivada. Note que isso envolve
 considerar P(x,y) como um polinômio em x cujos coeficientes são
 polinômios em y. Os livros de Curvas Algébricas em geral, vão falar
 deste parágrafo apenas (provar que a Resultante realmente faz a
 eliminação das variáveis, como calcular, como que os discriminantes
 têm a ver com resultantes).
 
 Exemplinho: Seja P(x,y) = (x^2 + x y^2 + x y + 2y + 2) = x^2 + (y^2 +
 y)*x + (2y + 1). Chame a = 1, b = (y^2 + y), c = (2y + 1). Nesse caso,
 o discriminante é o usual, ou seja, b^2 - 4*a*c = (y^4 + 2y^3 + y^2)
 - 8y - 4. As raízes disso dão os y máximos e mínimos locais.
 Dá pra ver tudo isso com o WolframAlpha
 
 http://www.wolframalpha.com/input/?i=+x^2+%2B+%28y^2+%2B+y%29*x+%2B+%282y+%2B+1%29+%3D+0
 http://www.wolframalpha.com/input/?i=%28y^4+%2B+2y^3+%2B+y^2%29+-+8y+-+4+%3D+0
 http://www.wolframalpha.com/input/?i=discriminant%28+x^2+%2B+%28y^2+%2B+y%29*x+%2B+%282y+%2B+1%29%2C+x%29
 
 
 Voltando ao seu problema, o WA dá o discriminante:
 http://www.wolframalpha.com/input/?i=discriminant%28%28-s^2y%2Bx^2y%2By^3%29Cos[A]%2B%28-s^2x%2B2sx^2-x^3

RE: [obm-l] funcao implicita e geogebra

2013-10-04 Por tôpico Luís Lopes
Sauda,c~oes, 

Problema resolvido mas desconheço a teoria que 
fornece a equação para calcular o máximo e mínimo 
da curva. 
 
Máximo e mínimo valor de y de la curva: Hay que resolver en y la ecuación: 



-s^4 + 10 s^2 y^2 + 2 y^4 + s^2 (s^2 - 12 y^2)Cos[2A] + (-6 s^3 y + 8 s 
y^3)Sin[2A] = 0

Foi falado num determinante,  sem maiores detalhes. 

E numa mensagem recente daqui (problema de tangência 

numa elipse) falou-se de um determinante também. 
Deve-se tratar da mesma coisa. 



Sds, 
Luís 


From: qed_te...@hotmail.com
To: obm-l@mat.puc-rio.br
Subject: [obm-l] funcao implicita e geogebra
Date: Thu, 3 Oct 2013 21:30:41 +







Sauda,c~oes, 

Não conheço muito do Geogebra e talvez alguém aqui 
possa me ajudar. 

O que segue é uma investigação sobre o problema de 
construir o triângulo ABC dados A,a+b,h_a. Algebricamente 
somente pois temos uma cúbica nos cálculos. 

Consegui descobrir que um lugar geométrico para o vértice A 
é dado pela cúbica 

===
La ecuación del lugar geométrico es:

(-s^2y+x^2y+y^3)Cos[A]+(-s^2x+2sx^2-x^3+2sy^2-xy^2)Sin[A]=0 
===
com s=a+b. 

Gostaria de calcular o valor máximo de y (que aqui representa a 
altura do triângulo) e assim ter somente uma solução.

No que segue s=10, Cos[A]=11/14 e Sin[A]=5sqrt(3)/14. 

Fui pro Geogebra e entrei os pontos B=(0,0) e A'=(10,0), bem como 
a função implícita do locus. 

(-10² y + x² y + y³) 11 / 14 + (-10² x + 2 (10) x² - x³ + 2 (10) y² - x y²) 5 
sqrt(3) / 14 = 0




Em seguida tracei a reta h_a=y=4.5 e obtive as duas interseções 
(vértices A) que interessam: A1 = (-0.38975, 4.5)


e A2 = (6.73216, 4.5)




. 
Daí C1 = (3.83061, 0)


e C2 = (5.26770, 0). Ou seja, obtive dois triângulos 
satisfazendo as condições. 

Até aqui tudo bem. Agora quero saber o valor máximo de y ou o ponto 
onde a derivada de 





(-10² y + x² y + y³) 11 / 14 + (-10² x + 2 (10) x² - x³ + 2 (10) y² - x y²) 5 
sqrt(3) / 14 = 0




se anula. Chamei esta equação de CC (para closed curve).

Voltei pro Geogebra e tentei usar o comando implicitderivative e ele me 
retornou 



CC'(x, y) = (15sqrt(3) x² - 200sqrt(3) x + 5sqrt(3) y² + 500sqrt(3) - 22x y) / 
(-10 sqrt(3) x y + 200sqrt(3) y + 11x² + 33y² - 1100)



Não sei bem o que estou fazendo agora. Ou mesmo como continuar sem o 
Geogebra. Alguém pode me explicar, continuar a investigação 
ou me dar o ponto de máximo? 

Obrigado. 

Sds, 
Luís 


  
--

Esta mensagem foi verificada pelo sistema de antivírus e 

 acredita-se estar livre de perigo.   
-- 
Esta mensagem foi verificada pelo sistema de antivírus e
 acredita-se estar livre de perigo.



RE: [obm-l] funcao implicita e geogebra

2013-10-04 Por tôpico Luís Lopes
Sauda,c~oes, 

Li errado. Não é determinante e sim discriminante. 

Continuo sem saber como calcular a equação que fornece 
os pontos extremos (max e min) da curva mas talvez a 
teoria se encontre nos livros que tratam das Curvas Algébricas Planas. 

Sds, 
Luís 


From: qed_te...@hotmail.com
To: obm-l@mat.puc-rio.br
Subject: RE: [obm-l] funcao implicita e geogebra
Date: Fri, 4 Oct 2013 20:02:40 +




Sauda,c~oes, 

Problema resolvido mas desconheço a teoria que 
fornece a equação para calcular o máximo e mínimo 
da curva. 
 
Máximo e mínimo valor de y de la curva: Hay que resolver en y la ecuación: 



-s^4 + 10 s^2 y^2 + 2 y^4 + s^2 (s^2 - 12 y^2)Cos[2A] + (-6 s^3 y + 8 s 
y^3)Sin[2A] = 0

Foi falado num determinante,  sem maiores detalhes. 

E numa mensagem recente daqui (problema de tangência 
numa elipse) falou-se de um determinante também. 
Deve-se tratar da mesma coisa. 

Sds, 
Luís 


From: qed_te...@hotmail.com
To: obm-l@mat.puc-rio.br
Subject: [obm-l] funcao implicita e geogebra
Date: Thu, 3 Oct 2013 21:30:41 +







Sauda,c~oes, 

Não conheço muito do Geogebra e talvez alguém aqui 
possa me ajudar. 

O que segue é uma investigação sobre o problema de 
construir o triângulo ABC dados A,a+b,h_a. Algebricamente 
somente pois temos uma cúbica nos cálculos. 

Consegui descobrir que um lugar geométrico para o vértice A 
é dado pela cúbica 

===
La ecuación del lugar geométrico es:

(-s^2y+x^2y+y^3)Cos[A]+(-s^2x+2sx^2-x^3+2sy^2-xy^2)Sin[A]=0 
===
com s=a+b. 

Gostaria de calcular o valor máximo de y (que aqui representa a 
altura do triângulo) e assim ter somente uma solução.

No que segue s=10, Cos[A]=11/14 e Sin[A]=5sqrt(3)/14. 

Fui pro Geogebra e entrei os pontos B=(0,0) e A'=(10,0), bem como 
a função implícita do locus. 

(-10² y + x² y + y³) 11 / 14 + (-10² x + 2 (10) x² - x³ + 2 (10) y² - x y²) 5 
sqrt(3) / 14 = 0




Em seguida tracei a reta h_a=y=4.5 e obtive as duas interseções 
(vértices A) que interessam: A1 = (-0.38975, 4.5)


e A2 = (6.73216, 4.5)




. 
Daí C1 = (3.83061, 0)


e C2 = (5.26770, 0). Ou seja, obtive dois triângulos 
satisfazendo as condições. 

Até aqui tudo bem. Agora quero saber o valor máximo de y ou o ponto 
onde a derivada de 





(-10² y + x² y + y³) 11 / 14 + (-10² x + 2 (10) x² - x³ + 2 (10) y² - x y²) 5 
sqrt(3) / 14 = 0




se anula. Chamei esta equação de CC (para closed curve).

Voltei pro Geogebra e tentei usar o comando implicitderivative e ele me 
retornou 



CC'(x, y) = (15sqrt(3) x² - 200sqrt(3) x + 5sqrt(3) y² + 500sqrt(3) - 22x y) / 
(-10 sqrt(3) x y + 200sqrt(3) y + 11x² + 33y² - 1100)



Não sei bem o que estou fazendo agora. Ou mesmo como continuar sem o 
Geogebra. Alguém pode me explicar, continuar a investigação 
ou me dar o ponto de máximo? 

Obrigado. 

Sds, 
Luís 


  
--

Esta mensagem foi verificada pelo sistema de antivírus e 

 acredita-se estar livre de perigo.   
--

Esta mensagem foi verificada pelo sistema de antivírus e 

 acredita-se estar livre de perigo.   
-- 
Esta mensagem foi verificada pelo sistema de antivírus e
 acredita-se estar livre de perigo.



[obm-l] funcao implicita e geogebra

2013-10-03 Por tôpico Luís Lopes



Sauda,c~oes, 

Não conheço muito do Geogebra e talvez alguém aqui 
possa me ajudar. 

O que segue é uma investigação sobre o problema de 
construir o triângulo ABC dados A,a+b,h_a. Algebricamente 
somente pois temos uma cúbica nos cálculos. 

Consegui descobrir que um lugar geométrico para o vértice A 
é dado pela cúbica 

===
La ecuación del lugar geométrico es:

(-s^2y+x^2y+y^3)Cos[A]+(-s^2x+2sx^2-x^3+2sy^2-xy^2)Sin[A]=0 
===
com s=a+b. 

Gostaria de calcular o valor máximo de y (que aqui representa a 
altura do triângulo) e assim ter somente uma solução.

No que segue s=10, Cos[A]=11/14 e Sin[A]=5sqrt(3)/14. 

Fui pro Geogebra e entrei os pontos B=(0,0) e A'=(10,0), bem como 
a função implícita do locus. 

(-10² y + x² y + y³) 11 / 14 + (-10² x + 2 (10) x² - x³ + 2 (10) y² - x y²) 5 
sqrt(3) / 14 = 0




Em seguida tracei a reta h_a=y=4.5 e obtive as duas interseções 
(vértices A) que interessam: A1 = (-0.38975, 4.5)


e A2 = (6.73216, 4.5)




. 
Daí C1 = (3.83061, 0)


e C2 = (5.26770, 0). Ou seja, obtive dois triângulos 
satisfazendo as condições. 

Até aqui tudo bem. Agora quero saber o valor máximo de y ou o ponto 
onde a derivada de 





(-10² y + x² y + y³) 11 / 14 + (-10² x + 2 (10) x² - x³ + 2 (10) y² - x y²) 5 
sqrt(3) / 14 = 0




se anula. Chamei esta equação de CC (para closed curve).

Voltei pro Geogebra e tentei usar o comando implicitderivative e ele me 
retornou 



CC'(x, y) = (15sqrt(3) x² - 200sqrt(3) x + 5sqrt(3) y² + 500sqrt(3) - 22x y) / 
(-10 sqrt(3) x y + 200sqrt(3) y + 11x² + 33y² - 1100)



Não sei bem o que estou fazendo agora. Ou mesmo como continuar sem o 
Geogebra. Alguém pode me explicar, continuar a investigação 
ou me dar o ponto de máximo? 

Obrigado. 

Sds, 
Luís 


  
-- 
Esta mensagem foi verificada pelo sistema de antivírus e
 acredita-se estar livre de perigo.



[obm-l] RE: [obm-l] Re: [obm-l] RE: [obm-l] Construção Geométrica (triângulos) ITA 1989

2013-07-01 Por tôpico Luís Lopes
Sauda,c~oes, oi Sergio, 
No google triangle construction given H_a,W_a,O aparecem outras soluções e 
comentários. 
Qual a fonte da sua construção ? 

Abs, Luis 

Date: Mon, 1 Jul 2013 09:58:53 -0300
Subject: [obm-l] Re: [obm-l] RE: [obm-l] Construção Geométrica (triângulos) ITA 
1989
From: sergi...@smt.ufrj.br
To: obm-l@mat.puc-rio.br

Caros,
Complementando entao a resposta do Luís Lopes,aqui vai a solução do problema:


ANÁLISE DO PROBLEMA:
Seja M a projeção de O na reta suporte de DH.Supondo a solução do problema 
conhecida,seja M´ a interseção de OM com a circunferência circunscrita.


Por uma análise angular simples é possível concluir queAOM' = (A + 2B) [ou (A 
+ 2C)], de modo que OAM' = OM'A = (C-B)/2[ou (B-C)/2]. Assim, AM' é a própria 
bissetriz interna do ângulo A
no triângulo desejado ABC, e, por isso mesmo, D pertence a AM'.
No triângulo AOM', com OA = OM', seja P1a altura do vértice O relativa ao lado 
AM'. Assim, temos duas propriedades
que nos permitem determinar o ponto P1:
(i) Como AOM' é isósceles, OP1 é perpendicular a AM' (e a AD).Assim, OP1D = 90 
graus, de modo que P1 pertence à circunferência de diâmetro OD.

(ii) Como AOM' é isósceles, P1 é o ponto médio de AM'. Como OM' é paralelaa AH 
(ambas são perpendiculares à reta suporte de DH), P1pertence à reta paralela a 
essas duas retas (OM' e AH) passando pelo ponto médio
de HM.

CONSTRUÇÂO
(i) trace a circunferência C1 de centro O1 e raio OO1, onde O1 é o ponto médio 
de OD,determinando a interseção M (ponto médio do lado BC) sobre a reta suporte 
de DH.
(ii) trace a perpencidular p à reta suporte de DH pelo ponto médio P de 
MH,determinando sobre C1 a(s) interseção(ões) P1 (e P2).(iii) prolongue DP1, 
determinando o vértice A sobre a perpendicular a DH por H.
(o prolongamento de DP2 gera uma outra solução para o vértice A).(iv) trace a 
circunferência de centro O e raio OA circunscrita ao triângulo,determinando os 
outros dois vértices B e C sobre a reta suporte de DH.

OBS 1: É possível ter 0/1/2 solução(ões) para o vértice A,dependendo se a reta 
p não-intercepta/tangencia/é-secante a C1.
OBS 2: Os vértices B e C podem ser intercambiados.

Abracos,sergio

2013/6/26 Luís Lopes qed_te...@hotmail.com




Sauda,c~oes, oi Sergio, 
Sim, continuo na lista. 
Caiu no ITA, foi? Bom saber. 
Gosto mesmo destes problemas. Vou mandar em seguida mais 
um, que acabo de conhecer. Problema (presente) de grego. 
===Eu não consegui, mas obtive a solução na internet(a qual envio numa próxima 
mensagem).
===Fico curioso. Conseguir como? Com o Google?? E e e ??? 
Para construir o triângulo, precisamos conhecer um resultado fundamental: a 
bissetriz ASa é bissetriz também do ângulo 
HaAO. 
Outro fato, esse elementar: a reta (A , Ha) é perpendicular â reta (Ha , Sa). 
Ultima dica: pense num circulo e numa reta espertos . 

Valeu Sergio pelo problema. 
Abs, Luis 

Date: Wed, 26 Jun 2013 08:01:02 -0300
Subject: [obm-l] Construção Geométrica (triângulos) ITA 1989

From: sergi...@smt.ufrj.br
To: obm-l@mat.puc-rio.br

Essa é em homenagem ao Luís Lopes e ao E. Wagner
(não sei se ainda acompanham a lista):
Construa o triângulo ABC dados em posição:. o pé Ha da altura do vértice A em 
relação ao lado BC.
. a interseção Sa da bissetriz do ângulo A com o lado BC.. o circuncentro O 
do triângulo.
Eu não consegui, mas obtive a solução na internet
(a qual envio numa próxima mensagem).
Abraço,sergio


--

Esta mensagem foi verificada pelo sistema de antivírus e 

 acredita-se estar livre de perigo.   
--

Esta mensagem foi verificada pelo sistema de antivírus e 

 acredita-se estar livre de perigo.





--

Esta mensagem foi verificada pelo sistema de antivírus e 

 acredita-se estar livre de perigo.   
-- 
Esta mensagem foi verificada pelo sistema de antivírus e
 acredita-se estar livre de perigo.



[obm-l] presente de grego 2

2013-07-01 Por tôpico Luís Lopes
Sauda,c~oes, 
Da mesma lista do anterior. 

[APH]In a triangle are given: a, A and the ratio |b-c| / h_a = m/n  (where 
h_a is the altitude from A) Prove that it has an Euclidean construction. 
(Ref.: Parartema, Nov. 1939, p.393)[Luis]: Again, I have no idea.  May I have 
a hint ?  Thanks. Dear Luis,Whenever we have an angle [so sin(of_that_angle) = 
sin(of_the_sum_of_the_other_two)] a good idea is to express the sin or cosof 
the difference of the other angles by the data of theproblem.If the equation we 
get is of = 2 degree it is possibleto have an euclidean construction.But how 
to make that construction geometrically is another story
Alguém saberia construir o triângulo dados a, A and the ratio |b-c| / h_a = m/n 
 ? 
Ou expressar sin(B-C) em função destes dados ? 
Abs, Luis 
  
-- 
Esta mensagem foi verificada pelo sistema de antivírus e
 acredita-se estar livre de perigo.



[obm-l] RE: [obm-l] Construção Geométrica (triângulos) ITA 1989

2013-06-26 Por tôpico Luís Lopes
Sauda,c~oes, oi Sergio, 
Sim, continuo na lista. 
Caiu no ITA, foi? Bom saber. 
Gosto mesmo destes problemas. Vou mandar em seguida mais um, que acabo de 
conhecer. Problema (presente) de grego. 
===Eu não consegui, mas obtive a solução na internet(a qual envio numa próxima 
mensagem).===Fico curioso. Conseguir como? Com o Google?? E e e ??? 
Para construir o triângulo, precisamos conhecer um resultado fundamental: a 
bissetriz ASa é bissetriz também do ângulo HaAO. 
Outro fato, esse elementar: a reta (A , Ha) é perpendicular â reta (Ha , Sa). 
Ultima dica: pense num circulo e numa reta espertos . 
Valeu Sergio pelo problema. 
Abs, Luis 

Date: Wed, 26 Jun 2013 08:01:02 -0300
Subject: [obm-l] Construção Geométrica (triângulos) ITA 1989
From: sergi...@smt.ufrj.br
To: obm-l@mat.puc-rio.br

Essa é em homenagem ao Luís Lopes e ao E. Wagner(não sei se ainda acompanham a 
lista):
Construa o triângulo ABC dados em posição:. o pé Ha da altura do vértice A em 
relação ao lado BC.
. a interseção Sa da bissetriz do ângulo A com o lado BC.. o circuncentro O 
do triângulo.
Eu não consegui, mas obtive a solução na internet
(a qual envio numa próxima mensagem).
Abraço,sergio


--

Esta mensagem foi verificada pelo sistema de antivírus e 

 acredita-se estar livre de perigo.   
-- 
Esta mensagem foi verificada pelo sistema de antivírus e
 acredita-se estar livre de perigo.



[obm-l] Presente de Grego

2013-06-26 Por tôpico Luís Lopes
Sauda,c~oes, 
Este problema apareceu nesta semana, numa outra lista da qual faço parte. 
To construct triangle if are given:1. the length of the altitude from A2. the 
length of the int. bisector of A3. the length of the orthogonal projection of 
the side BCon the external bisector of AReference: It is an old problem I found 
in a Greek math. magazine(Euclid, June 1969, p. 405)APH
Ou seja, construir ABC dados ha, da, a'. 
Dica facil: construir o triângulo AHaDa e o ponto Ea. Da e Ea s~ao os pés das 
bissetrizes interna e externa. 
Dica dificil: M é o ponto médio de AEa. N é o ponto médio de B'C'=a'. 
Abs, Luis 
  
-- 
Esta mensagem foi verificada pelo sistema de antivírus e
 acredita-se estar livre de perigo.



[obm-l] Prove - desigualdade

2013-05-31 Por tôpico Luís Eduardo Háteras
Sejam x,y,z números reais positivos tais que 1/(1+x) +  1/(1+y) +  1/(1+z) = 2. 
Prove que 8xyz = 1.  

[obm-l] equacao exponencial

2013-05-14 Por tôpico Luís Lopes

Sauda,c~oes, 

Pediram a minha ajuda no problema abaixo. 
Se sair truncado para alguns, o problema é: 

O número de pontos comuns aos gráficos das funções definidas por $y=e^x$ e $y= 
- \ln |x|$,  
$x\neq0$, é:
Como vocês sempre têm uns comentários espertos que me escapam, 
aguardo suas respostas. O email veio com a resposta, que cortei. 

Agora um pedido meu: gostaria de ter as respostas, não somente o número delas. 
Um Maple qualquer dá isso. Obrigado. 

Abs, 
Luís 

Caro Luis,
Gostaria de sua ajuda para a seguinte questão:
O número de pontos comuns aos gráficos das funções definidas por e , , é:

a) .
b) .
c) .
d) .
e) nenhuma das anteriores.
 RESPOSTA: 
??

[obm-l] A,a,h_a construir triangulo

2013-05-12 Por tôpico Luís Lopes
Sauda,c~oes, 

O problema de construir o triângulo ABC dados A,a,h_a 
é bem conhecido. Aparece como a,h_a,R no livro do Wagner 
de Construções Geométricas e é resolvido pelo Sérgio Lima Netto 
É o problema 28 do Capítulo 1. 

A construção que conheço e a mesma que vejo sempre nos livros 
usa o arco capaz de A sobre o segmento BC=a. O que vale dizer, 
o círculo circunscrito e seu raio R. Talvez por ser a mais  
natural??, evidente?? 

Isso se conhecemos e sabemos construir o arco capaz. Mas NÃO sei. 

Pensei então numa outra construção. Começo construindo o ângulo A. 
Assim B e C pertencem às retas que formam o ângulo A. E já sei também 
um lugar geométrico para H_a, pé da altura. Falta construir a reta 
suporte do lado a. Isso eu não sei. 

Como terminar a construção? 

Abs, 
Luís 


  

[obm-l] soma trigonometrica

2012-12-10 Por tôpico Luís Lopes







Sauda,c~oes, 

No último número da Eureka  


http://www.obm.org.br/export/sites/default/revista_eureka/docs/Eureka35.pdf 
www.obm.org.br/export/sites/default/revista_eureka/docs/Eureka35.pdf


encontrei na página 54  o problema 147. 

O problema é: mostrar que para n\geq 2 

S_n = \sum_{k=0}^{n-2} \frac{(-1)^k}{\sin\Bigl(\frac{(2k+1)\pi}{4n-2}\Bigr)} = 
n + \frac{(-1)^n - 1}{2} 

n=2  S_2=2

n=3  S_3=\csc(\pi/10) - \csc(3\pi/10) = 2. 

n=4  S_4=\csc(\pi/14) - \csc(3\pi/14) +  \csc(5\pi/14) = 4. 

Alguém (da Eureka?) poderia falar alguma coisa dessa fórmula? Tipo  

1) referência de onde foi tirada. 
2) outra demonstração. Seria possível por indução? Por recorrência? 
3) seria um caso particular de uma fórmula mais geral? 

Felicito o Michel Martins (autor da demonstração apresentada) pela 
demonstração. 

Abraços, 
Luís 



  

RE: [obm-l] Calcular o mdc (333...3, 333...3)

2012-12-09 Por tôpico Luís Lopes

Sauda,c~oes, 

Então quero calcular mdc entre (x^10 - 1) e (x^8 - 1), onde x=10^10.
Esta questão foi respondida pelo Buffara em 30/3/2007. 
Seja mdc(m,n)=d. Então mdc(x^n-1,x^m-1)=x^d-1. 

Off topic. 
Fiquei há uns dez dias sem receber as msgs da lista, mesmo as que eu 
mandava. Mas numa outra conta de email elas apareciam. Ontem recebi 
muitas msgs, acho que todas que havia ficado sem receber. E algumas 
(como as do Artur e Eduardo e outras) vinham com uma advertência do 
filtro do hotmail como msgs suspeitas. E vejo que o Artur e Eduardo enviam 
as mesmas msgs mais de uma vez. Devem estar com o mesmo problema. 
Suspeito que o site da obm-l esteja contaminado. 

Abraços, 
Luís 



From: marconeborge...@hotmail.com
To: obm-l@mat.puc-rio.br
Subject: RE: [obm-l] Calcular o mdc (333...3, 333...3)
Date: Sat, 8 Dec 2012 22:42:08 +





Muito legal!
 
Date: Wed, 5 Dec 2012 02:48:28 -0200
Subject: Re: [obm-l] Calcular o mdc (333...3, 333...3)
From: wgapetre...@gmail.com
To: obm-l@mat.puc-rio.br

O 1o numero é (10^100 - 1)/3, enquanto o 2o é (10^80 - 1)/3. Obviamente eu 
posso ignorar esse 1/3 aí, e depois dividir a resposta que eu achar por 3.
Então quero calcular mdc entre (x^10 - 1) e (x^8 - 1), onde x=10^10.

Então eu percebo que x^2 - 1 divide ambos (se eu não percebesse, eu sempre 
poderia fazer a divisão euclidiana deles). Dividindo tudo por x^2 - 1 fica:
p(x) = x^8 + x^6 + x^4 + x^2 + 1 e q(x) = x^6 + x^4 + x^2 + 1. Então eu percebo 
que p(x) - x^2*q(x) = 1 (novamente se eu não percebesse eu faria a divisão...). 
Então acabou pq se alguém divide p(x) e q(x) para algum x, então também divide 
1. Logo esses caras são primos entre si. Assim o mdc original fica (x^2-1)/3 = 
10^20 - 1



2012/12/4 Pedro Chaves brped...@hotmail.com





Colegas da Lista,

Como calcular o mdc (a, b) , sendo a = 333...3  (100 dígitos iguais a 3) e b = 
333...3 (80 dígitos iguais a 3)?


Abraços do pedro Chaves

_-




  

[obm-l] RE: PROBLEMA

2012-12-05 Por tôpico Luís Lopes

Sauda,c~oes, 

Não tenho recebido e conseguido mandar msgs para a lista. 

Esta vai como teste. Se receberem, continuem a ler. 

Luís 

From: qed_te...@hotmail.com
To: obm-l@mat.puc-rio.br
Subject: FW: PROBLEMA
Date: Mon, 3 Dec 2012 12:44:01 +





Sauda,c~oes, 

Mais uma vez, peço a ajuda de vocês para uma resposta 
mais completa e interessante. 

Espero que o anexo passe. Se não, alguém poderia colocá-lo 
disponível em algum site? 


Subject: PROBLEMA
Date: Mon, 3 Dec 2012 08:17:24 -0300








Prezado Luís, 



Gostaria de sua ajuda para o problema(conforme 
arquivo PDF anexo). 
Se possível, gostaria que justificasse porque os demais 
itens são verdadeiros. 


Abraços, 

FERNANDO FORTALEZA-CE. 

PS: E eu gostaria de saber também por que o item falso é falso. 
Obrigado. 

Abraços, 
Luís 



  

[obm-l] PROBLEMA

2012-12-05 Por tôpico Luís Lopes

Sauda,c~oes, 

Mais uma tentativa de ver esta mensagem chegar na lista. 

O Bernardo já respondeu numa troca de emails particular. 

Esta mensagem deve ser lida de baixo para cima. 

Abraços, 
Luís 



O arquivo está em 
http://www.escolademestres.com/download/questao_luis_lopes_20121205.pdf 

 Date: Wed, 5 Dec 2012 09:59:41 -0500
 Subject: Re: [obm-l] RE: PROBLEMA
 From: bernardo...@gmail.com
 To: qed_te...@hotmail.com
 
 2012/12/5 Luís Lopes qed_te...@hotmail.com:
  Oi Bernardo,
 
  Não consigo me comunicar com a lista. Mas pelo jeito a
  lista (você) recebe(u) minhas msgs.
 Estranho.
 
  Tenho recebido somente algumas msgs da lista. Num outro
  email (outra conta) acho que recebo todos. Pode ser problema
  com o hotmail também, sei lá.
 Pode ser...
 
  Tudo começou com o email abaixo, que mandei pra lista:
 Pode ser uma questão de PDFs. Sei lá. Pode ser mesmo esse PDF em particular.
 
 Para responder a pergunta: todas as afirmações são verdadeiras, é
 claro. O problema da d é que a projeção estereográfica envia
 círculos passando pelo polo Norte em retas do plano + ponto no
 infinito. Mas como f está definida apenas na esfera - polo Norte,
 nenhuma dessas circunferências serve, porque f não aplica o polo norte
 em lugar nenhum. Logo, as circunferências que você *pode* usar são
 realmente mandadas em círculos do plano.
 
 Ah, óbvio que isso é um problema de geometria complexa, mas fica muito
 mais legal retirar todo o contexto e fazer uma questão absurda de
 vestibular. Pena, né.
 -- 
 Bernardo Freitas Paulo da Costa

From: qed_te...@hotmail.com
To: obm-l@mat.puc-rio.br
Subject: FW: PROBLEMA
Date: Mon, 3 Dec 2012 12:44:01 +





Sauda,c~oes, 

Mais uma vez, peço a ajuda de vocês para uma resposta 
mais completa e interessante. 

Espero que o anexo passe. Se não, alguém poderia colocá-lo 
disponível em algum site? 


Subject: PROBLEMA
Date: Mon, 3 Dec 2012 08:17:24 -0300








Prezado Luís, 



Gostaria de sua ajuda para o problema(conforme 
arquivo PDF anexo). 
Se possível, gostaria que justificasse porque os demais 
itens são verdadeiros. 


Abraços, 

FERNANDO FORTALEZA-CE. 

PS: E eu gostaria de saber também por que o item falso é falso. 
Obrigado. 

Abraços, 
Luís 

  

[obm-l] Re: [obm-l] FerroVelho Matemático - Reativando

2012-10-19 Por tôpico Luís Junior
Boa idéia! Já favoritei.

2012/10/19 terence thirteen peterdirich...@gmail.com

 Olá pessoas!

 Estou reativando meu antigo site no Google, FerroVelho Matemático.
 Minha ideia é postar alguns problemas que andei resolvendo em tempos
 de olimpíada. Acaso gostem e queiram sugerir algo, fica a dica!

 https://sites.google.com/site/ferrovelhomatematico/

 --
 /**/
 神が祝福

 Torres

 =
 Instru�ões para entrar na lista, sair da lista e usar a lista em
 http://www.mat.puc-rio.br/~obmlistas/obm-l.html
 =



Re: [obm-l] sair da lista

2012-10-16 Por tôpico Luís Junior
Ahh Rita, fica vai... vou me sentir sozinho e com saudades!

2012/10/16 Rita Gomes rcggo...@terra.com.br


 Quero sair da lista



[obm-l] series telescopicas

2012-09-24 Por tôpico Luís Lopes

Sauda,c~oes, 

Recebi o seguinte email: 


Boa tarde! Caro Prof. Luís Lopes, estou interessado em saber mais sobre
 as séries telescópicas, sendo mais especifico sobre a origem do termo 
telescópica, o porquê desse nome e como ele surgiu para definir esse 
tipo de série, desde já agradeço a atenção e estarei aguardando 
ansiosamente uma resposta. Obrigado!


O porquê eu até acho que sei: aquele tubo da luneta se espicha e 
se recolhe sobre si mesmo. Daí a imagem com as séries telescópicas. 

Mas origem do termo, como e quando surgiu . ?? 

[]'s 
Luís 

  

[obm-l] RE: [obm-l] Re: [obm-l] Geometria(Construção(2))

2012-09-12 Por tôpico Luís Lopes

Sauda,c~oes, oi Ralph, 

Gostei da sua construção do triângulo. Eu começaria 
traçando o Â. Depois a bissetriz etc. Mas a sua 
construção é melhor. 

No quadrilátero APIQ o PÎQ = 180 - Â. Então o 
tamanho do arco PQ não seria 180 - Â ?? 

Para que a construção funcione, precisamos que Q esteja entre A e C, isto é, 
que br.sin(Â/2)AQ = r.cot(Â/2). Logo, br.cot(Â/2) 

[]'s 
Luís 



Date: Tue, 11 Sep 2012 13:47:40 -0300
Subject: [obm-l] Re: [obm-l] Geometria(Construção(2))
From: ralp...@gmail.com
To: obm-l@mat.puc-rio.br

1) Considere o círculo de diâmetro P1P2. Ele contém o vértice A do quadrado... 
Mas, melhor ainda, pense na diagonal AC! Como ela é a bissetriz do ângulo 
P1AP2, então ela passa pelo ponto D1, médio do arco P1P2 daquele círculo (veja 
figura anexa, viva Geogebra!), que é DETERMINADO A PARTIR DE P1 e P2!

Analogamente, você pode encontrar D3, médio do arco P3P4 do círculo de diâmetro 
P3P4. Como D1D3 será a diagonal do quadrado, você pode intersectar esta 
diagonal com os círculos para achar A e C.

Agora repita para B e D, e acabou!
(Se alguém quiser o Geogebra da construção, mando por E-mail -- acho que a 
lista não aceitaria o anexo)
2) Este é bem mais simples:
i) Desenhe o círculo inscrito, marque nele um arco PQ de tamanho 180-2A.ii) 
Trace as tangentes ao círculo por P e Q, intersecte-as, este é o ponto A (note 
que PÂQ=Â, de fato).iii) Agora é só marcar b (a partir de A) em cima da reta AQ 
para achar o ponto C...
iv)... e traçar a tangente ao círculo por C para achar o ponto B (sobre AP).
Para que a construção funcione, precisamos que Q esteja entre A e C, isto é, 
que br.sin(Â/2)

Abraço,   Ralph
P.S.: Note que há um outro arco P1P2! O que aconteceria se a gente escolhesse 
D1 como médio desse OUTRO arco, assim como D3, D2 e D4? :) :) :)

2012/9/10 marcone augusto araújo borges marconeborge...@hotmail.com








1)Os pontos P1,P2,P3,P4 pertencem aos lados consecutivos de um quadrado 
ABCD.Construa com régua e compasso o quadrado.Justifique sua construção.
 
 
 .P1
 
   .P2

 
 
 
.P3  
 

 .P4 
 
 2) Construa o triangulo ABC conhecendo o angulo A,o lado b e o raio r do 
círculo inscrito.Justifique.   

  

  

[obm-l] RE: [obm-l] Re: [obm-l] RE: [obm-l] Re: [obm-l] Socorro em geometria (construçã o)

2012-09-11 Por tôpico Luís Lopes




Sauda,c~oes, oi Rogerio, 

Vc tem razão. Vi isso depois de ter mandado a mensagem. 
O problema é quase igual ao que originou estas mensagens: 

===
Construir o trapézio ABCD conhecendo a soma das bases AB + CD = s, 
as diagonais AC = p e BD = q e o lado AD = a. Justifique.   
   
===
Esse foi resolvido. 

Imaginei um com a diferença das bases: 

Construir o trapézio ABCD conhecendo a diferença das bases AB - CD = s, 
as diagonais AC = p e BD = q e o lado DA = d. 

Mas este problema não é construtível com RC. 

Como não consegui imaginar uma construção, mandei-o para uma outra lista. 
E obtive a seguinte resposta: 

=
from my calculations it seems that AB=a must satisfy the cubic equation

2 a^3 - 3 s a^2 + (2 d^2 - p^2 - q^2 + s^2) a - d^2 s + q^2 s = 0, 

hence a is not always constructible.
=

[]'s 
Luís 


Date: Tue, 11 Sep 2012 11:07:13 -0300
Subject: [obm-l] Re: [obm-l] RE: [obm-l] Re: [obm-l] Socorro em geometria 
(construçã o)
From: abrlw...@gmail.com
To: obm-l@mat.puc-rio.br

Ola' Luis,
com apenas 3 elementos conhecidos ( a diferenca das bases, e as diagonais ) o 
terceiro problema esta' indeterminado.
[]'s
Rogerio Ponce



Em 8 de setembro de 2012 12:45, Luís Lopes qed_te...@hotmail.com escreveu:





Sauda,c~oes, 

 Por T passe uma paralela a AT. Com centro em A desenhe um arco de raio a. Esse
Typo. Por C  

Ou seja, TC é o transformado de BD pela translação do vetor DC. 

Outras construções: construir um trapézio ABCD, conhecendo-se as bases 

AB=a, CD=c, e 

1) os dois lados, BC=b e AD=d. 

2) as diagonais AC=p e BD=q. 

3) a diferença das bases AB - CD = d, as diagonais AC = p e BD = q. 

1) e 2) são problemas do tipo: dado o comprimento e a direção de um segmento, 

apoiá-lo em duas linhas (curvas). 

Apesar de parecido com o que foi acabado de ser construído, não consegui 
resolver o 3). Ou seja, não descobri a translação conveniente. 

Considero a disposição dos vértices como em 


http://en.wikipedia.org/wiki/Trapezoid

Luís 


 From: saldana...@pucp.edu.pe

 To: obm-l@mat.puc-rio.br
 CC: 
 Subject: [obm-l] Re: [obm-l] Socorro em  geometria (construçã o)
 Date: Sat, 8 Sep 2012 06:51:19 -0500

 
 
 
 Contrói o triângulo ACT com lados AC=p, CT=q e AT=s.
 
 Por T passe uma paralela a AT. Com centro em A desenhe um arco de raio a. Esse
 arco vai cortar à paralela (tem 2 soluções). Chame de D a ese ponto de corte.

 Por D traçe uma paralea a CT, o ponto de corte entre essa paralea e AT será o
 ponto B do trapecio ABCD procurado.
 
 Julio Saldaña
 
 
 -- Mensaje original ---
 De : obm-l@mat.puc-rio.br

 Para : obm-l@mat.puc-rio.br
 Fecha : Sat, 8 Sep 2012 03:09:59 +
 Asunto : [obm-l] Socorro em  geometria (construçã o)
 
 

 
 
 Construir o trapézio ABCD conhecendo a soma das bases AB +CD = s,as diagonais
 AC = p e BD = q e o lado AD = a.Justifique.   

 
 __

 Si desea recibir, semanalmente, el Boletín Electrónico de la PUCP, ingrese a:
 http://www.pucp.edu.pe/puntoedu/suscribete/
 

 =
 Instruções para entrar na lista, sair da lista e usar a lista em
 http://www.mat.puc-rio.br/~obmlistas/obm-l.html

 =
  


  

[obm-l] RE: [obm-l] Re: [obm-l] Socorro em geometria (construçã o)

2012-09-08 Por tôpico Luís Lopes

Sauda,c~oes, 

 Por T passe uma paralela a AT. Com centro em A desenhe um arco de raio a. Esse
Typo. Por C  

Ou seja, TC é o transformado de BD pela translação do vetor DC. 

Outras construções: construir um trapézio ABCD, conhecendo-se as bases 
AB=a, CD=c, e 

1) os dois lados, BC=b e AD=d. 

2) as diagonais AC=p e BD=q. 

3) a diferença das bases AB - CD = d, as diagonais AC = p e BD = q. 

1) e 2) são problemas do tipo: dado o comprimento e a direção de um segmento, 
apoiá-lo em duas linhas (curvas). 

Apesar de parecido com o que foi acabado de ser construído, não consegui 
resolver o 3). Ou seja, não descobri a translação conveniente. 

Considero a disposição dos vértices como em 

http://en.wikipedia.org/wiki/Trapezoid

Luís 


 From: saldana...@pucp.edu.pe
 To: obm-l@mat.puc-rio.br
 CC: 
 Subject: [obm-l] Re: [obm-l] Socorro em  geometria (construçã o)
 Date: Sat, 8 Sep 2012 06:51:19 -0500
 
 
 
 Contrói o triângulo ACT com lados AC=p, CT=q e AT=s.
 
 Por T passe uma paralela a AT. Com centro em A desenhe um arco de raio a. Esse
 arco vai cortar à paralela (tem 2 soluções). Chame de D a ese ponto de corte.
 Por D traçe uma paralea a CT, o ponto de corte entre essa paralea e AT será o
 ponto B do trapecio ABCD procurado.
 
 Julio Saldaña
 
 
 -- Mensaje original ---
 De : obm-l@mat.puc-rio.br
 Para : obm-l@mat.puc-rio.br
 Fecha : Sat, 8 Sep 2012 03:09:59 +
 Asunto : [obm-l] Socorro em  geometria (construçã o)
 
 
 
 
 Construir o trapézio ABCD conhecendo a soma das bases AB +CD = s,as diagonais
 AC = p e BD = q e o lado AD = a.Justifique.   

 
 __
 Si desea recibir, semanalmente, el Boletín Electrónico de la PUCP, ingrese a:
 http://www.pucp.edu.pe/puntoedu/suscribete/
 
 =
 Instruções para entrar na lista, sair da lista e usar a lista em
 http://www.mat.puc-rio.br/~obmlistas/obm-l.html
 =
  

[obm-l] FW: solicitação

2012-07-27 Por tôpico Luís Lopes

Sds, 
Alguém pode ajudar? Obrigado. 
Luis



Subject: solicitação
Date: Fri, 27 Jul 2012 08:14:16 -0300





Em uma escola 95% das pessoas tem 1,62 m ou mais e 8% 1,62,m ou menos. Calcule 
o % de quem tem 1,62 .Solicito uma ajuda nesta questão. 
  

  

[obm-l] FW: solicitação

2012-07-27 Por tôpico Luís Lopes

Sds, 
Reenviando. 
From: qed_texte@hotmail.comTo: ob...@mat.puc-rio.brSubject: FW: 
solicitaçãoDate: Fri, 27 Jul 2012 12:12:47 +Sds, Alguém pode ajudar? 
Obrigado. LuisSubject: solicitaçãoDate: Fri, 27 Jul 2012 08:14:16 -0300Em uma 
escola 95% das pessoas tem 1,62 m ou mais e 8% 1,62,m ou menos. Calcule o % de 
quem tem 1,62 .Solicito uma ajuda nesta questão. 
  

[obm-l] FW: PROBLEMAS..... de concurso??

2012-06-25 Por tôpico Luís Lopes

Sauda,c~oes, 
Me mandaram os problemas abaixo com o gabarito. Que tirei para ver as respostas 
justificadas de vocês, sempre melhores e mais espertas do que as minhas. 
Faço isso por 3 razões:
1) para me ajudarem; 2) para dar uma melhor resposta ao Fernando; 3) para tirar 
a lista do silêncio e moviment'a-la um pouco. 
[ ]'s Lu'is 

 
Prezado Luis,
 
Gostaria de sua ajuda para as seguintes 
questões:


1)Se 
dois trinômios do 2º grau possuem as mesmas raízes então:
a) eles são 
necessariamente iguais.
b) eles assumem necessariamente um mínimo ou um 
máximo no mesmo ponto.
c) eles diferem por uma constante.
d) suas 
concavidades são de mesmo sentido.
e) nenhuma das anteriores.
R. letra  a letra d é f'acil de ser eliminada. hum a letra a também
 
2)Dados três pontos no plano cartesiano, não colineares e com abscissas 
distintas duas a duas, o número de funções quadráticas que podem ser 
encontradas 
de maneira que esses pontos pertençam aos seus gráficos é:
a) 
0  b) 
1  c) 
2  d) 
3
R.letra .? 
  

[obm-l] conjectura sobre colinearidade

2012-05-29 Por tôpico Luís Lopes

Sauda,c~oes, 

Retomo uma (muito) velha mensagem. 

Continuo ao final das mensagens (nada como um 
bom sistema de arquivamento). 

O Claudio Buffara ainda acessa a lista? 

-Mensagem Original-
De: Claudio Buffara claudio.buff...@terra.com.br
Para: obm-l@mat.puc-rio.br
Enviada em: sábado, 6 de março de 2004 01:42
Assunto: Re: [obm-l] conjectura sobre colinearidade


  Sauda,c~oes,
 
  Seja dado o triangulo AP_0Q_0 .
 
  Em AP_0 e AQ_0 marcamos P_0Q_i,
  e Q_0P_i  tais que P_0Q_i = Q_0P_i = m_i,
  i = 1,2,  e m_i  m_{i+1} (todos diferentes
  entre si). Unimos P_0P_i  e  Q_0Q_i,
  obtendo a interseção R_i.
 
  Conjectura: os R_i são colineares.
 
  Como provar? Qual a teoria que suporta
  tal resultado? Teorema de Desargue?
 
  Se a conjectura vira um teorema, temos
  uma solução para os problemas
  A,a+b,a-c  e  A,a-b,a-c.
Typo: A,a+b,a+c  e  A,a-b,a-c.

 
  []'s
  Luís
 
 
 Oi, Luis:

 A conjectura eh verdadeira. Veja a seguir...

 Considere o triangulo APQ e vetores unitarios u e v 
 nas direcoes PA e QA, respectivamente. Se |PA| = b e 
 |QA| = c, entao teremos que o vetor PQ serah bu - cv.

 Sejam Q' sobre AP e P' sobre AQ tais que |PQ'| = |QP'| = m.
 Entao, PQ' = mu  e  QP' = mv.

 PP' = bu - cv + mv = bu + (m-c)v
 QQ' = mu - (bu - cv) = (m-b)u + cv

 Interseccao de PP' e QQ' == existem x e y reais tais que:
 R = PQ + x*QQ' = y*PP' ==
 bu - cv + x*((m-b)u + cv) = y*(bu + (m-c)v) ==
 (b + (m-b)x - by)u + (-c + cx - (m-c)y)v = 0

 Como o triangulo APQ eh nao degenerado, u e v sao L.I.
 Assim:
 (m-b)x - by = -b
 cx - (m-c)y = c

 Resolvendo este sistema, obtemos:
 x = b/(b+c-m)  e  y = c/(b+c-m)

 O ponto de interseccao serah:
 R = y*PP' = c(bu + (m-c)v)/(b+c-m)

 dR/dm = bc(u + v)/(b+c-m) = multiplo de um 
 vetor constante (u + v) == ao se variar m, 
 R percorre uma linha reta == CQD

 Um abraco,
 Claudio.


Tudo muito bem. Hoje sei (ver a mensagem de 08/03/04) 
http://www.mat.puc-rio.br/~nicolau/olimp/obm-l.200403/msg00294.html
que a conjectura é verdadeira usando feixes anarmônicos, 
razões anarmônicas (cross-ratios), raios homólogos 
e feixes perspectivos. 
Tudo isso é geometria projetiva com uma abordagem cearense. 

A reta suporte dos pontos R (interseção de raios homólogos) 
é o eixo da perspectiva. 
Ou da homologia. E o vértice A, o centro da homologia. 

Acompanhei a demonstração do Buffara e acho que tá 
tudo ok. Ou quase. 

Empaquei aqui. 

 O ponto de interseccao serah:
 R = y*PP' = c(bu + (m-c)v)/(b+c-m)
Ok. Então R=f(m), como esperado. 

 dR/dm = bc(u + v)/(b+c-m) = multiplo de um 
 vetor constante (u + v) == ao se variar m, 
 R percorre uma linha reta == CQD
Não seria dR/dm = bc(u + v)/(b+c-m)^2 ?? 

E como R percorre uma linha reta? 

dR/dm = k(u+v) múltiplo de um vetor constante 
(u+v). Onde k=bc/(b+c-m)^2. 

Para R percorrer uma reta, k não teria que ser 
independente de m também??? 

Gostaria de comentários, correção, confirmação 
sobre o final da mensagem do Buffara. 

Obrigado. 

Luís 

  

[obm-l] questoes EPCAR

2012-05-22 Por tôpico Luís Lopes




Sauda,c~oes, 

Alguém pode resolver? Obrigado. 

Abs, 
Luís 



01) EPCAR - 2007 A dá a B tantos reais quantos B possui e A dá a C 
tantos reais quantos C possui. 
Depois, B dá a A e a C tantos reais 
quantos cada um possui e C, finalmente, faz a mesma coisa. 
Se no final, 
terminam todos com 16 reais e sabendo que C começou com 50% de B mais um
 real, 
então A começou com 

a) 24 reais  
 c) 28 reais b) 26 reais
   d) 30 reais 


02) EPCAR - 2007 
Uma loja colocou um CD à venda por R$ 28,00 a unidade. 
Como não atraiu 
muitos compradores, resolveu baixar o preço para um 
número inteiro de 
reais. Com isso, vendeu o restante do estoque 
que não era superior a 50 
unidades, por R$ 377,00. Com base nisso, 
o número n de unidades do CD 
restante no estoque é um número 
cuja soma dos algarismos vale 
 
a)
 6  c) 11 b) 9  
d) 15   




  

RE: [obm-l] Soma

2012-04-22 Por tôpico Luís Lopes

Sauda,c~oes, 





Não seria 





1 + 2*2^1 + 3*2^2 + 4*2^3 + ... + n*2^(n-1) ? 





O termo geral a_k pode ser escrito como a_k = (a_1 + (k-1)r)q^{k-1} 





com a_1=r=1 e q=2. 

Temos então uma progressão aritmético-geométrica cuja fórmula fechada 
para a soma S_n = \sum_{k=1}^n a_k  é

S_n = \frac{a_1(q^n - 1)}{q-1} + \frac{qrA}{(q-1)^2} com 

A = 1 - nq^{n-1} + (n-1)q^n 

Substituindo a_1=r=1 e q=2, vem:

S_n = (n-1)2^n + 1

Trato deste assunto, bem como de material que complementa o que o 
Nehab mandou recentemente sobre Médias no Manual de Progressões. 

Abs, 
Luís 


 Date: Sun, 22 Apr 2012 08:08:53 -0300
 From: smo...@terra.com.br
 To: obm-l@mat.puc-rio.br
 Subject: Re: [obm-l] Soma
 
 Ops... cometi o velho erro de trocar o sinal. resposta final deve ser 
 (n-1).(2^n) - 1


 =
 Instruções para entrar na lista, sair da lista e usar a lista em
 http://www.mat.puc-rio.br/~obmlistas/obm-l.html
 =
  

  1   2   3   4   5   >